Você está na página 1de 74
Exercicios* 1. Os dados abaixo referem-se & taxa de creatinina na urina de 24 horas (mg/100 ml), em uma amostra de 36 homens norma. Indivin@ Creat. | tnaiv.ne Creat. | Indiv.nt Creat. | Indiv.nt Great ot 181 10 1,08; 19 154 266 oe 181 " 1.66 20 138 ee 03 188 2 1.52 2 147 ogee) 04 149 3 440 2 4.73 3t 140 05 1.87 4 1183 2 1.60 tae 06 2.18 8 1.22 24 143 3482 07 1,46 16 1.48 2 1.58 3487 08 1.88 ” 1.43 28 188 3186 os 1,76 18 1.48 27 1.26 3 202 1.1. Organize uma tabela de freqiiéncias, adotando classes iguais, de modo que a primeira seja 1,00 1,15. 1.2, Determine as freqiiéncias absoluta, relativa e acumulada relativa (com tres decimais) de cada classe. 1.3. Determine a percentagem de observagées: a) no intervalo 1,75 F—1,90 b) menores do que 1,45 ©) no intervalo 1,30 1,60 4) iguais ou maiores que 1,90 1.4, Supondo que esta amostra representa a populacdo de homens normais, qual a probabilidade de que um individuo dessa populacdo apresente uma taxa de creatinina: a) igual ou maior do que 2,05? b) entre 1,45 € 1,60 exclusive? ©) menor do que 1,30? 2. Desenhe um histograms para os dados do exercicio 1, usando a freqiiéncia relativa. + Respostas ao final da lista. 186. Side M. Callegari-facques Caviruios 2 £3 3. Certo bidlogo esté medindo o dano ambiental em uma reserva ecolégica. Em seis locais dessa reserva, determinou um escore de dano (em graus), obten- do os dados a seguir. 3.1, Caleule a média, a mediana, a variancia ¢ 0 desvio padrdo e coloque unida- des nessas estatisticas. 3.2. Repita os célculos incluindo o local L7, no qual foi observado um escore de dano igual a 4 (sugestao: use as formulas alternativas para o célculo da variancia). Local: loo 13 ASG ano) Gaus): a) ot ee 4, Os dados a seguir referem-se a idade (em anos) de um grupo de 20 criangas. Obtenha a média, o intervalo modal e 0 desvio padrao. i Hdade f iF 3 7 3k 5 5 sk 7 4 | mk 9 3 9k 1 5. Calcule a média, a mediana, a amplitude, a variancia ¢ 0 desvio padrao para a seguinte amostra de dados de hematécrito: areas eed ASG 52a SOs ayaa ol 6. Observe as duas séries a seguir, de dados de glicemia (nivel de glicose no sangue, em mg/100 mL), comparando médias, amplitudes e desvios padrio (DP). Indique qual das duas medidas de variagao é a mais eficiente para medir a dispersdo dos dados ao redor da média. Indique em que série essa dispersio é maior. Amostra 1 Amostra 2 x t x t 34 1 aa 2 92 1 ‘Media = 100 92 0 Média = 100 98 2 pp. 96 2 100 3 100 1 108 2 108 4 108 1 108 1 6 1 118 1 7. Na série 2 do Exercicio 6, a média da idade dessas pessoas foi 25 anos € 0 desvio padrao, 19 anos. Que caracteristica, nessa série, apresenta maior va- riabilidade: a idade ou a glicemia? = Bioestatistica 187 Cariruto 4 Em 113 ninhos da tartaruga Dermochelys coriacea, o ntimero médio (+ desvio padrao) de filhotes nascidos vivos foi 53 (#26) e o tempo médio de incuba- ‘io, 70 (#8) dias (Morisso e Krause, 2001). Use as propriedades da curva normal para avaliar se a distribuicéo dessas varidveis pode ter gaussiana (Sugestio: calcule os extremos esperados para cada variavel, supondo uma distribuigdo gaussiana.) 9. Determine a area da curva normal correspondente aos valores de s indicados: 9.1, # entre Oe 2 9.2. smaior do que 1,5 9.3. s menor do que -1,0 9.4, sentre-1,5 € 0,98 9.5. sentre-2,3 e-1,2 10. Que valores de s que limitam as éreas indicadas a seguir? 101 10.2 | | osr7 013 | | | | 10.3 104 | 0,989) | 0,025 fi | 11. _Emdeterminada populacéo, a taxa de hemoglobina no sangue tem distribuigo normal, com média igual a 16 g/100 ml. e desvio padrao de 1,2 g/100 mL. 11.1. Que proporcio de individuos tem taxa menor do que 17,87 11.2. Que proporcao de individuos tem taxa maior do que 18,4? 11.3. Qual sua opiniao sobre um nivel de hemoglobina igual a 14g/100 mL quan- do comparado com a média? 11.4, Quantas pessoas tém valor entre 14,8 e 16,6 em uma amostra de 2.500 | individuos, pressupondo que, na amostra, a distribuiggo dos dados é a mes- ma da populacao? 12. Em certa populacio, a estatura dos homens tem distribuicdo normal, com média igual 172 em e desvio padrao igual 10 cm. 12.1. Que percentagem de homens tem estatura inferior a 160 cm? 488 Sidia M. Callegar-Jacques 12.2. Qual a probabilidade de que um homem dessa populacio tenha estatura entre 175 € 185 em? 12.3. Quais sio as estaturas esperadas para os 8% mais altos da populacdo? 13. Preocupado com o desempenho em matemética dos alunos da 8 série, certa escola deseja proporcionar aulas extras para aqueles que tém mais dificl dade nessa matéria. Os professores tém condiedes de atender 10% do total de alunos da 8* série. Sabendo que as notas nessa série tém distribuigéo normal, com média igual a 7,0 e desvio padrao igual a 1,0, determine a nota abaixo da qual o aluno receberd aulas extras. 14, Em determinado concurso, havia 600 candidatos para 120 vagas. Realizada a prova, o mimero médio de acertos foi 70, com desvio padrao de 5. Qual 0 niimero minimo de acertos para que win candidato se classifique, sabendo que esta varidvel apresentou distribuicéo normal? Cantrutos 5, 6 £7 15. Suponha que o nivel de colesterol total no sangue de pessoas sadias tem media igual a 200 mg/dL. e desvio padrao igual a 50 mg/dl. Calcule os valo- res-limite da regio de ndo-significdncia (a = 0,05) para as médias de amos- tras de n = 16. Como se deve interpretar uma média igual a 230 mg/dL, obtida em uma amostra de 16 pessoas? 16. O comprimento da concha de certa espécie de molusco tem média igual a 31,2 mm e desvio padrao igual a 3,0 mm. Em um grupo de 14 individuos dessa espécie, obtidos no sul da América do Sul, foi obtida uma média igual 27,8 mm (Ducattie Pitoni, 1995). Verifique se esse valor desvia-se signifi- cativamente da média para a espécie (a. = 5%). 17. Desenhe uma curva de distribui¢éo normal para valores de x, com média igual a 100 e desvio padrao igual a 20. Esquematize sobre ela as curvas de distribuicio amostral de médias obtidas em amostras de n=16 en = 4, tira- das desta populacio. A que conclusao vocé chega? 18. Cem pacientes foram estudados quanto & taxa de fenilalanina no soro. A média obtida foi 1,3 mg/100 mL. e supés-se de que tais individuos seriam uma amostra casual da populacao de individuos normais, onde a média é 1,4 0 desvio padrao, 0,32 mg/100 mL. Considerando-se um nivel de significan cia de 0,01, os dados esto de acordo com a suposicéo feita? 19. Suponha que nas tartarugas da espécie Chrysemys d’Orbignyi, a glicemia tenha média igual a 86 mg/100 ml, e desvio padrao-igual a 8 mg/100 mL. Um biélogo operou & tartarugas dessa espécie, retirando-Ihes a hipéfise. Apés determinado tempo, mediu a glicemia nos animais operados ¢ obteve média igual a 37 e desvio padrao igual a 12 mg/100 mL, (Foglia e colaboradores, 1955, modificado). Que pode ele afirmar sobre o efeito da remogéo da hipo- fise na glicemia desses animais (a = 0,01)? Bioesrarisrica 189 20. Em certa espécie de plantas ornamentais, o comprimento médio das semen- tesé de 6 mm. Em uma amostra de sete sementes de uma nova variedade, os valores obtidos foram os indicados a seguir. Compare as duas variedades entre si quanto ao comprimento das sementes (a = 0,05). x (mm): 6; 7,5; 7; 6,5; 8; 9; 8,5 21. Para fins de aproveitamento do solo, a vegetacio lenhosa pode ser eliminada or corte ou por corte seguido de queima dos galhos. Quando a limpeza de ‘um terreno é feita apenas por corte da vegetacio, o nivel de chumbo no solo, quatro anos apés 0 processo, é de 9,31 partes por milo (ppm). De uma rea que sofreu corte seguido de queimada, foram coletadas, 4 anos apés, 44 amostras de solo e determinado o contetido de chumbo. A média obtida foi 8,44 ppm e o desvio padrao foi 3,23 ppm (Girardi-Deiro, 1999). Analise estas informagGes e conclua sobre o efeito da queimada na quantidade desse metal no solo, usando a = 0,05. 22, Em homens adultos sauddveis, o nivel de hemoglobina, ao nivel do mar, tem média 16 g/100 mL. Os dados a seguir foram obtidos em uma amos- tra aleat6ria de 10 mulheres sauddveis, que vivem em uma praia gaticha, Teste (a = 0,05) a hipstese de que o nivel de hemoglobina é 0 mesmo em. homens e mulheres saudaveis. x(@/100 mL): 12 14 16 13 15 11 17 12 14 16 23, Dutra e colaboradores (1986) dosaram a concentracdo de fenilalanina (umol/ L) em 8 genitores de criancas com fenilcetomiria e obtiveram média igual a 158,5 e desvio padrao igual a 18,8. Suponha que a média em individuos normais é 79,4. O que podem concluir os pesquisadores (a = 0,01)? 24. O volume da tiredide foi medida em 46 criancas'com idade entre 6 ¢ 14 anos, da cidade de Passo Fundo, RS. A média nessa amostra foi 4,6 ml. e 0 desvio padrio, 1,4 ml. (Lisbéa ¢ colaboradores, 1996). Estime a média po- pulacional para essa varidvel, usando intervalos de 90% e 95% de confian- a. Discuta os resultados quanto A preciso das estimativas. 25. O contetido médio de material sélido em suspensdo na agua do rio R costu- ma ser de 205 mg/L. Uma coleta recente, em 9 pontos desse curso d’agua, forneceu os dados a seguir. Verifique se houve alteraco na quantidade de material sélido em suspensfo, para um nfvel de significancia de 0,05. x (mg/L): 210; 242; 226; 268; 251; 206; 218; 215; 207 (¥: 2275: 21,9) 26. 0 teor de cobre (ppm) foi medido em 43 plantas que cresceram em uma rea que sofreu um processo de corte seguido de queima do material lenho- so. A média foi 15,2 ppm e o desvio padrao foi 4,04 (Girardi-Deiro, 1999) Qual o verdadeiro teor de cobre na vegetacio que cresce em dreas que sofre- ram esse tipo de manejo? 27. Foi estudada a quantidade (x10°/mm®) de plaquetas no sangue de 24 pa- cientes com trombose venosa (Robinson, 1974). A média obtida no sangue venoso foi 282 e o desvio padrao 70. 490 Sidia M. Callegari-Jacques 27.1. Construa 0 intervalo de 95% de confianca para a média da populacio de pacientes com esse problema. 27.2. A adesividade plaquetéria foi também medida nessa amostra, A média foi 14% e 08 valores variaram entre 0 € 53%. Voct diria que a distribuigao desses dados ¢ gaussiana? 28. 0 intervalo de 95% de confianca que estima 2 média de uma populacio é 35,39-44,61. Esse intervalo foi caleulado em uma amostra de 61 individuos, com a média igual a 40. Qual o valor do desvio padrio nesta amostra? Carirutos 8 £9 29. Onivel sangiiineo de glicose foi medido em 53 pacientes diabéticos com pro- teintiria (excrecio alta de proteina na urina) e 64 diabéticos sem tal caracte- ristica, A média t DP no primeiro grupo foi 214 + 74 e no segundo, 168 + 63 (Gross e colaboradores, 1993). Compare estes dois grupos de diabéticos quan- to & glicemia e conclua sobre os resultados obtidos, usando o nivel 0,01 de significdncia, 30. Freitas e Lessa (1984) compararam duas amostras de um roedor silvestre conhecido como “tuco-tuco” (Ctenomys torquatus) quanto a varias medidas do crnio, As amostras diferiam quanto ao mtimero de cromossomos: a amostra ‘A (obtida em Laranjal, Rio Grande do Sul) tinha 44 cromossomos e a B (do ‘Taim, Rio Grande do Sul) tinha 46. Pode-se afirmar que existe relacdo entre co niimero cromossémico e as medidas indicadas a seguir (a = 0,05)? Media + OP Vardvel Laranjal (724 animals) ‘Taim (0-8) Comprimento | 46,1215 4535168 Largura da maneibula 353214 34217 31. Os dados a seguir referem-se ao grau de conforto (valores mais altos, mais con orto) no uso de dois tipos de pipetas de laboratério. O primeiro tipo foi experi- ‘mentado por 5 pessoas e o segundo por outras 5, totalizando 10 usudrios. Com que tipo de pipeta os usudrios sentem-se mais confortdveis (cr = 0,01)? TpoA: 95 1 3 4 2 Tipo B: 5 8 9 7 6 32, Tesche e colaboradores (1984) informam 0 comprimento ¢ o didmetro de ovos de jacaré-do-papo-amarelo (Caiman latirostris) obtidos em dois locais: no Zoo- légico de Sapucaia do Sul, RS, e no Jardim Zoolégico do Rio de Janeiro, RU. Compare os dois conjuntos de dados por um teste estatistico, usando a = 0,01. Tamme Roa ei aw Senge) aaa wae 8 Digmetro (nm) 45,75 0,88 38.340,68 2 Bioestasisica 194 al 33. Modificagbes nos niveis de meta-hemoglobina (MHb) e sulfo-hemoglobina (SHib) no sangue podem ocotrer por aco de substéncias téxicas produzidas Por inckistrias. Naoum e colaboradores (1984, dados modificados) mediram a MHb e a SHb em uma amostra de moradores da Vila Parisi, um bairro cereado pela maioria das indiistrias de Cubato (SP), e em 38 controles, habitantes de uma cidade praticamente sem poluigao industrial. Analise os dados apresentados a seguir e conclua. Use « = 0,05 em todos os testes. Media OP Variaver Vila Parsi (9=18) Contole (n=33) gla Metachemogiobina 81923,17 6,63=208 4925 Sulo-hemoglobina 3,052.32 1194 # 1.04 4921 34. Tendo interesse em estudar os efeitos de determinada dieta alimentar sobre © aumento do peso corporal em cobaias adultas, um investigador tomou uma amostra de 9 cobaias. Determinou seus pesos antes e trés meses apés a administragio da nova dieta. Com os dados a seguir, analise o efeito da nova dieta, para a = 0,05. Antes: 54 61 SO 74 «79 «SB SS 49 63 Depois: 57 66 53 73 82 58 56 53 63 35. Um médico mediu a pressdo arterial de 10 hipertensos antes e depois de tomarem, durante uma semana, um novo medicamento destinado a reduzir a presséo arterial. Com os dados seguintes, em cmHlg, verifique se a nova droga tem 0 efeito desejado (realize um teste unilateral com a = 0,05). Antes: ph es) 3D 3 ty ) FH St) Depoi BG) 19) 9) 16) 15) 1300 16) a0) 19 36. Desejando determinar o efeito do medicamento M sobre o colesterol plasmé- tico, certo investigador realizou um experimento com 25 pacientes. Cada paciente recebeu um placebo e a droga em estudo em dois diferentes mo- ‘mentos, tendo seus niveis de colesterol medidos apés cada administracio. A diferenca média entre medidas, calculada na forma “placebo menos M”, foi 1,8 mg/100 mL. O desvio padrao das diferencas foi 2,24. Qual o efeito desse medicamento para a = 0,01? Cavtrutos 10 #11 37. Cinco pessoas, que se submeteram a uma mesma cirurgia de joelho, usa- ram dois instrumentos de avaliacdo para indicar o nivel de dor 12 horas apés a operacdo. A seguir esto os escores de dor de cada pessoa, em cada instrumento, 37.1. Desenhe o diagrama de dispersio dos pontos, considerando como x os re- sultados obtidos no primeiro instrumento de avaliacio e como y, 0s do se- gundo. =a 192 Sidia M. Callegari-Jacques 37.2. Calcule o coeficiente de correlacéo entre os escores determinados nos dois instrumentos. 37.3. Verifique se o coeficiente de correlacio é estatisticamente significativo, para Pessoa: A B C DE Dor(instrumento): 8 6 4 3 4 Dor (Instrumento2): 9 7 4 4 6 38. Rocha e Pena (1987) dosaram os niveis de alfa-fetoproteina em 30 amostras de iquido amnidtico humano e observaram uma correlacdo de -0,66 coma idade gestacional e de -0,45 com a quantidade de fibronectina neste liqui- do. Teste a significdncia dos coeficientes obtidos e conclua sobre a correla. cio entre as varidveis, para um nivel de significdncia de 0,01. 39. Visando estudar 0 ciclo reprodutivo de lagartos pequenos da espécie Liolae- ‘mus oceipitalis, que ocorrem no litoral do Rio Grande do Sul, Verrastro e Krause (1999) mediram o volume testicular (VT) de exemplares do sexo masculino coletados entre abril de 1986 e marco de 1988, e calcularam o coeficiente de correlagio entre VT ¢ a temperatura na data da coleta. 0 valor obtido foi r = -0,75 (P<0,01). Interprete o resultado. 40. Represente graficamente as seguintes equagées, arbitrando valores para x (embre-se de que bastam dois pontos para desenhar uma reta). 40.1 40.2. ¥ 05+3X — 40.3.Y=4-X -1-2x 40.4. Y= -2 + 3X 41, Foram estudadas 9 criancas com o objetivo de verificar se existe regressio a capacidade pulmonar sobre a idade. Os dados esto apresentados a se- guir. Desenhe 0 grafico de disperséo dos pontos experimentais. Estime a reta de regressio de y sobre x e ajuste-a sobre os pontos. Teste a significan- cia da regressio e interprete o resultado obtido (a = 0,05). Idade(anos): 94 5 6 7 8 9 10 il 12 Capital): 0,7 09 12 13 13 LS 17 19 21 42. Doses crescentes de calcério foram adicionadas a um solo écido e depois determinou-se a percentagem de anomalias encontradas em células germ nativas de trigo plantado nesse solo. Obtenha uma reta de regressao da percentagem de anomalias sobre a quantidade de calcério no solo, teste & significéncia do coeficiente de regressao para a = 0,01 e conclua. Quantidade de caledrio: o 1 2 3 4 5 % de anomalias celulares: 0) 27) oe 23) 1B ee) 43. Como objetivo de descrever as equagées que caracterizam o tipo de cresci ‘mento em uma espécie de caranguejos do litoral norte paulista, Mantelatto e Fransozo (1994) estudaram a relagio entre a largura do abdémen (mm) € Bioestatistica 193 a da carapaga (mm) nesta espécie. Na andlise, foi usada transformagao lo- garftmica (base e) nas duas varidveis. Os resultados a seguir referem-se ape- nas a dados de 21 fémeas jovens. O coeficiente de correlacio entre as duas varidveis foi 0,88 (P<0,001) e a equaco de regressio foi: Inlargura do abdomen) = -2,59 + 1,25 In(largura da carapaca). No teste de significén- cia da regressio, o erro padrao foi 0,15. 43.1, Interprete o coeficiente de correlacio. 43.2. Teste o coeficiente de regressao ¢ interprete a equagio de regressao. 43.3. Diz-se que existe isometria quando diferentes partes do corpo crescem de forma proporcional. Nesse caso, o coeficiente de regresséo entre as varié- veis deve ser 1. Verifique se os dados contradizem a existéncia de isometria entre as varidveis estudadas em fémeas jovens dessa populacio, para a=0,05 (Sugesto: use B = 1). Ganfruvo 12 44. Desenhe um gréfico de colunas para os dados a seguir. Descreva em que aspectos esse grafico distingue-se de um histograma, Classitcagio de 121 recém-nascids itrcas, do sexo masculn, confome 0 Indie de desenvolimento intra-uterine (Heidi, 1992) PIG AIG. ais Total 2 (16%) 97 (80.2%) 22 (18.2%) 121 PIG, AIG e GIG: pequeno, adequado © grande para a dade gestacional, respectivamente, Capiruios 18 w 4 45, Varios estudos mostram que, na espécie humana, aproximadamente 40% dos abortos espontaneos devem.se a alteracGes cromoss6micas (Borges-Os6- rio e Robinson, 2001). Duas mulheres comentam entre si que tiveram um aborto esponténeo. Qual a probabilidade de que os dois sejam devidos a alteragées cromossémicas? 46. A probabilidade de nascer uma crianga com a sindrome de Down, se a mae tem mais de que 44 anos, é de 1/30 (Thompson e Thompson, 1988). Qual a proba- bilidade de que uma mulher gravida, de 45 anos, vena a ter uma filha normal? 47. 0 suprimento de energia elétrica de um hospital provém das fontes A e B, cujas probabilidades de falhar so, respectivamente, de 0,03 e 0,1. Qual a probabilidade de: 47.1, Nenhuma falhar? 47.2. Uma das duas falhar? 473. As duas falharem? 48. A fenilcetontiria ¢ determinada por um gene recessivo autossomico. Um ca- sal sadio tem um filho com essa doenca. 48.1, Qual a probabilidade de que os préximos dois filhos que pretendem ter se- Jam normais? 194 Sidia M. Callegari-Jacques 48.2. Nasceu o segundo filho ¢ ele é normal. Nesse caso, a probabilidade de ter- ceiro ser normal passa a ser 1/2? Por qué? 49. Na regido metropolitan de Porto Alegre, 32% das pessoas tém o tipo san- giiineo A, e 8%, Ay (Dornelles, 1998). Um estudante necessita de uma trans- fusio de sangue de qualquer tipo A. Trés de seus amigos se oferecem, mas no conhecem seu préprio grupo sangilineo. 49.1. Qual a probabilidade de que os trés tenham o tipo sangtiineo A? 49.2, Um dos trés colegas teve hepatite, no podendo portanto doar sangue. Con- siderando os dois restantes, qual a probabilidade de que ao menos um tenha o tipo sangilineo A? 50. Em determinada populacdo, foi aplicada uma vacina que costuma produzir imunizacio realmente efetiva em 9 casos entre 10. Qual a probabilidade de que em um grupo de sete pessoas: 50.1. Todas se imunizem? 50.2. Nenhuma se imunize? 50.3. Uma nao se imunize? 50.4. Pelo menos 6 se imunizem? 50.5. No maximo 4 se imunizem? 51.1. Desenhe um grafico de bastées para representar a varidvel p = % pessoas com defeitos visuais em uma amostra den = 12. A probabilidade de defel- tos desse tipo na populagio (P) € 0,5. 51.2. Repita o procedimento para uma populacio onde P 51.3, Repita o procedimento para uma populacéo onde P = 52, A espécie Drosophila willistoni é encontrada em frutos de Butia eriospatha (butid), na Rua Osvaldo G.Cruz, em Porto Alegre. Nessa populagao de mos- cas-das-frutas, 60% dos individuos apresentam a inversio cromossOmica E zo eromossomo IIL (Valente e colaboradores, 1993). Uma bidloga vai a essa rua e coleta individuos dessa espécie de insetos. Calcule a probabilidade de que, em uma amostra de 20 individuos coletados, 15 ou mais sejam porta- dores da inversio cromossOmica E no cromossomo IIL. Use inicialmente @ distribuigio binomial, depois a normal com e sem correcdo para continuida- de e compare os resultados obtidos pelos trés procedimentos. 53. Durante o periodo de margo/1990 a janeiro/1991, foram avaliados todos os pacientes admitidos na unidade de lactentes de alto risco de um hospital pedidtrico de Porto Alegre (Pinto e colaboradores, 1996). Dos 106 que apre~ sentavam uma doenca com componente genético, 14 (13,2%6) eram casos de doencas devidas a genes autoss6micos recessivos. Supondo que essa amos- tra possa ser considerada representativa, estime com 0,95 de confianca @ percentage verdadeira de casos de doencas devidas a genes deste tipo, na populagdo de recém-nascidos internados por distirbios genéticos em unida- des de alto risco, 54. Estime a percentagem de fumantes entre pessoas que tém trombose venose, com 95% de confianea, sabendo que & eram fumantes em uma amostra de Bivestattstica 195 25 pacientes (Robinson, 1974). Explique a razdo de se obter um intervalo ‘com to pouca preciso e proponha um procedimento para aumenté-la. 55. Certo grupo de ambientalistas decidiu realizar um campanha de cons- cientizacio sobre a selegdo de lixo reaproveitavel em uma comunidade na qual 30% dos domicflios selecionam o lixo, Se, ao final da campanha, 32 de 80 domicflios amostrados aleatoriamente estiverem selecionando 6 lixo, vocé conclui que houve mudanga de comportamento na comuni- dade (a = 0,05)? 56. Aprevaléncia de esofagite (inflamacao da mucosa do esBfago), no sexo mas- culino, € de 34% em pessoas que vivem em uma regio considerada de bai- xo risco para cfincer de es6fago (China ~ Jiaoxian) e de 83% no Ira, pais considerado de alto risco. Em um estudo envolvendo 60 homens gatichos, 26 tiveram diagnéstico positive para esofagite (Mufioz e colaboradores, 1987), Compare a proporcao de casos positivos nas gatichos com a da China e depois com a do Ird e elabore uma conclusdo (a = 0,05) 57. Michelon ¢ Moriguchi (1996) estudaram uma amostra de pessoas com 80 anos ou mais, da cidade de Veranépolis, RS. Foram encontrados niveis de colesterol total iguais ou superiores a 240 mg/dL. em trés dos 35 homens examinados (8,6%) e em 15 (23,1%) das 65 mulheres estudadas. £ correto afirmar, com base nessas informagoes, que a freqiiéncia de pessoas com ni- veis elevados (2 240) de colesterol total é maior em mulheres do que em homens idosos dessa populagao (c= 0,05)? 58. A freqiiéncia de plantas com mosaicismo cromossémico foi estudada em duas cultivares de trigo desenvolvidas no Rio Grande do Sul (Bodanese-Zanettini € colaboradores, 1993). Em 35 plantas da cultivar CNT-10, foram observa- as 20 plantas com mosaico (57,19), e em 32 plantas IAS-52, 5 eram plan- tas-mosaicos (15,6%). Compare as cultivares de trigo entre si quanto a essa caracteristica genética (a = 0,05) 59. ‘Trés pesquisadores avaliaram o mimero de ovos depositados pela borbo- leta “maria-boba” (Heliconius erato phyllis) em plantas de maraeujé (Pas- siflora misera) durante o ano de 1981 e conclufram que houve uma varia- cdo mensal na ovoposi¢ao (Romanowsky e colaboradores, 1985). Gom- prove essa conclusio com os dados a seguir, usando 0 método estatfstico apropriado. Mes: Mar Abr Mai Jun Jul Ago Set Out Nov (Total) Neovosdep: 17 4 28 39 3 1 0 1 32 (125) 60. Em determinada empresa, foram registrados os seguintes mimeros de aci- dentes de trabalho no més de agosto de 1998: 42.em segundas-feiras, 23 em tercas-feiras, 25 em quartas-feiras, 19 em quintas-feiras, 23 em sextas-feiras, 48 em sébados (manhi e tarde). Verifique se os acidentes de trabalho ocor- rem com 2 mesma freqiiéncia nos seis dias iteis da semana (a = 0,01). 198 Sidia M. Callegari-Jacques 61 62, 63, 64, 65, Em um experimento genético clissico realizado no infcio do século XX, H, Nilsson-Ehle cruzou plantas de trigo que produziam sementes vermelho-es- curas e plantas com sementes brancas ¢ obteve descendentes que produziam sementes de cores diferentes, nas seguintes proporcdes: 1/16 eram plantas que produziam gros vermelho escuro, 4/16 produziam sementes verme- has, 6/16 vermelho médio, 4/16 vermelho claro ¢ 1/16 eram plantas com sementes brancas. Em um cruzamento de igual natureza, certo agrdnomo obteve 6, 12, 29, 17 e 0 individuos de cada tipo, respectivamente, e propés a hipdtese de que estaria havendo, nas condicées de seu experimento, uma selecdo contra o trigo de graos brancos. Os dados apoiam a hipétese formu. ada (a = 0,05)? Na populagdo caucasdide, 70% das pessoas sentem o gosto amargo da PIC (feniltiocarbamida), enquanto 30% sio insensfveis a essa substancia. Supo- nha que em uma amostra de 240 caucasdides com problemas de tiredide, foram encontrados 144 sensiveis, Existe uma relagao entre problemas na tiredide e sensibilidade ao PTC (a = 0,05)? ‘0s dados seguintes referem-se & cor da pelagem em uma amostra de roedo- res de determinada espécie. Verifique se tal caracterfstica esté associada com ‘sexo (@=0,05). Machos: preto: 22 marrom: 13 manchado: 15 Fémeas: preta: 16 marrom: 17 manchada: 17 Com o objetivo de avaliar fatores de risco para o cfncer intra-epitelial da cérvice uterina, Soares (1998) estudou 43 casos com essa doenca e 63 mu: Iheres controles da populacdo de Porto Alegre. As tabelas a seguir apresen. tam dados relativos & idade de inicio das relagbes sexuais € a presenca do alelo DQB1*03, do sistema HLA, nessas mulheres. Teste separadamente a associacio entre cada uma destas variveis e 0 desenvolvimento da neopla sia em estudo (a = 0,05). ade de inicio da vida sexual Aisio <16__ 1720 >20 Total DQBIOS Outro Tote Cases, 7 2 2 43 Casos 3108 Contioles 16 —-33——14_—«G3_—Controles Pie Doll ¢ Bradford-Hill (1952) realizaram um extenso estudo sobre a etiolo- gia do céncer de pulmao em doentes ingleses. Uma das partes do estudo referiu-se & associacio entre esse carcinoma e o mimero de cigarros fi- mados por dia, Cada paciente com cancer havia sido pareado com outra pessoa do mesmo sexo e idade, que estava hospitalizada por outro pro- blema que nao cdncer de térax, vias aéreas superiores, labios ou outro ‘rgao que pudesse estar relacionado ao habito de fumar. A tabela a seguit apresenta os dados obtidas em homens. Conelua sobre a associagao men- cionada, para @ = 0,001. 66. 66.1, 66.2. CaptruLo 16 68. 68.1 Bioestatistica 197 P>0,02 12; P>0,10 112, Verrastro e Krause (1994) estudaram espécimens de Liolaemus occipitalis, pequeno lagarto da regio costeira do Rio Grande do Sul. Suponha que te- nham sido encontrados os valores a seguir, relativos ao comprimento rostro- anal (CRA, em mm) e ao peso (em ). Individuo 1 2 3 4 5 CRA(mm) 47 51 54, 59 62 Peso (g) 5,0 39 67 6,0 95 112.1, Desenhe o gréfico de dispersdo dos pontos, estime a reta de regressdo de ¥ (pes0) sobre x (comprimento rostro-anal) e ajuste a reta de regressio sobre os pontos observados, Desenhe 0 grafico dos residuos. Teste a significdncia do coeficiente de regresso (a = 0,05) e conclua. Em uma amostra maior de 115 fémeas, as pesquisadoras obtiveram a equa- ‘40: peso = -10,76 + 0,30 CRA, tendo relatado um valor P<0,001 para o teste de significdincia da regressio, Modifique (ou ndo) sua conclusio com base no resultado da nova amostra. 112.5. No mesmo conjunto de 115 fémeas, as autoras observaram uma relacdo estatisticamente significativa entre a taxa de crescimento e o comprimen- to rostro-anal, do tipo: taxa de crescimento = 14,2 ~0,20 CRA. Interprete o resultado, T 208 Sidia M. Callegari-Jucques 113. 0 comprimento do eranio foi estudado em duas amostras de roedores silves.. tres, uma de Tijuca e outra de Teresdpolis, Rio de Janeiro (Pessoa e Reis, 1991). Verifique se essa medida morfolégica esté relacionada com o local de coleta dos roedores, para « = 0,01. Tijuca: 24 média = 53,4 mm 1,48 mm Teresépolis: n=18 — média = 50,1 mm 2,19 mun 114. Certo professor submeten 9 criancas a um novo método de aprendizagem de leitura, enquanto e outras 7 aprenderam pelo método tradicional. Conelua sobre a eficigncia do novo método, para c = 0,05, usando os dados a seguir, que correspondem as notas obtidas em um teste de leitura igual para os dois grupos. ‘Método tradicional (n=7): 10 7 8 5 4 8 7 Novo método (n=9): 9 8 9 8 97 4 8 4 115,,A atividade de renina plasmética (ng/mlL/h) foi medida em 22 diabéticos e 16 individuos-controle (Azevedo, 1992; p. 84). Compare as amostras entre si usando um teste paramétrico, para alfa = 0,08. Amoata a a ot Diabaticos 2 36 28 Contioles 16 116. Na espécie de borboleta Heliconius erato, a proporgéo de individuos que apre- sentam manchas de cor creme nas asas posteriores € de 0,83 (Romanowsky e colaboradores, 1985). Admitindo que 53% das borboletas capturadas sdo ‘machos e que nao hé associacio entre sexo e presenca dessas manchas, qual a probabilidade de se capturar: 116.1. Uma fémea com manchas? 116.2. Uma fémea sem manchas e um macho com manchas, nesta ordem? 116.3. Duas fémeas e um macho, todos com manchas, nesta ordem? 117. Suponha que de todos os estudantes que ingressam no primeiro grau, apenas 40% se graduam na 8 série, Qual a probabilidade de que, dentre 6 estudan- tes da 18 série: 117.1, Os seis terminem 0 1° grau? 117.2, Apenas um termine o 12 grau? 117.3. Ao menos quatro terminem o 12 grau? 118. Um lote de aparethos de radio é considerado satisfatério, pela comissao de controle de qualidade, se néo for encontrado nenhum defeito em uma amos- tra de 5 rédios selecionados ao acaso da producéo. Determine a probabilids- de de um lote de aparelhos ser considerado satisfatério: 118.1. Se 10% de todos os aparethos do lote tém realmente algum defeito. 118.2. Se 5% de todos os aparelhos do ote tém realmente algum defeito, Eel Bioestatistica 209 119. Em certa populagio de colegiais, 20% tém problemas de visio. Para uma amostra de 10 criancas dessa populacéo, determine a probabilidade de que: 119.1. Exatamente cinco tenham problemas visuais. 119.2. Pelo menos cinco tenham problemas visuals. 119.3. Menos do que duas tenham problemas visuais. 120. Pinheiro (1989) entrevistou gestantes de dois hospitais de Porto Alegre quanto ‘a0 hébito de fumar. Em 1137 gestantes da Santa Casa de Misericérdia, 522 (45,9%) eram fumantes, enquanto 177 (29,6%) fumavam dentre as 597 es- ‘tudadas no Hospital de Clinicas. Compare os dois grupos de pessoas quanto ao hdbito de fumar e conclua, para a = 0,01 121. A aco de um inseto-praga costuma afetar 35% da producio de milho de certa fazenda, O fazendeiro, ento, plantou uma nova variedade de milho supostamente mais resistente. Na colheita ele obteve, em uma amostra de 100 pés selecionados ao acaso, 25% de perda por aco dessa praga. A nova variedade ¢ realmente mais resistente ao inseto (a = 0,05)? 122. Em certa localidade, a percentagem de motoristas envolvidos em zero, um ‘mais de um acidentes anuais € 65%, 30% e 5%, respectivamente. Uma com- panhia de seguros tomou uma amostra de 100 de seus segurados e encon- trou 66, 23 ¢ 11 individuos nessas categorias. Os dados so compativeis com as proporgdes observada nessa localidade (a = 0,05)? 123, Dornelles (1998) relata as freqiiéncias de diferentes fendtipos para o sistema MN em individuos caucasdides origindrios das regides nordeste, sudoeste ¢ sudeste do Estado do Rio Grande do Sul. Compare as regides entre si, usan- do 0 nivel 0,05 de significdncia. Regio Fenétipo MN TE 30 OSOSC~SE M 9 cy 19 un 126 7 30 N 49 26 4 Total 24 167 8 124, Em uma amostra aleatéria de 1598 escolares de Porto Alegre, foram encon- tradas 349 criancas (21,8%) com distirbios de escrita (Borges-Osério, 1985). Com base nessa’ amostra, estime, com 99% de confianga, a proporcao de escolares porto-alegrenses com esse problema. 125. Bau ¢ colaboradores (2001) estudaram dois grupos de aleoolistas gatichos do sexo masculino. No primeiro grupo, os problemas decorrentes do alcool comecaram ao redor dos 33 anos, enquanto no segundo, comecaram mais cedo (ao redor dos 23 anos). A tabela a seguir apresenta o mimero de pes- s0as com alucinacdes nos dois grupos. Compare os grupos entre si quanto & freqiiéncia de alucinagdes (a = 0,01). 20 Sidia M. Callegari-Jacques 126. 127. 128) 129. 130. 131. 132. Com alucinagées Sem alucinagses NV ‘Grupo t (problemas mais tarde) 10 38 8 Grupo 2 (problemas mais cedo) 67 2 93 Certo bidlogo estudou o efeito de um gradiente de luminosidade na distribui- cio de determinada graminea. Foram estabelecidas trés zonas de intensida- de luminosa, indicadas por I (menor luminosidade) a III (maior luminosida- de). Em uma amostra de 33 plantas, 6 foram encontradas na zona I, 12 na zona Il e 15 na zona Ill. A distribuicio dessa graminea depende do nivel de luminosidade (a = 0,05)? Para estudar 0 efeito de uma droga potencialmente antiemética no controle de vémitos pés-operatdrios, uma experiéncia foi realizada com 100 pessoas. Das 65 que receberam a droga, 22 tiveram vmitos, enquanto entre as 35 que receberam um placebo (substdncia inerte), 28 tiveram vémitos. Teste a eficdcia dessa droga para @ = 0,01 Certo fabricante de pasta de dentes quis testar a opiniéo dos consumidores de duas cidades sobre uma nova embalagem para seu produto. Na cidade A, foram entrevistadas 400 pessoas e 256 mostraram-se favoraveis & nova emba- lagem. Na localidade B, 110 foram favordveis em 200 entrevistados. Compa- re as cidades entre si quanto & opinido sobre a embalagem nova (a = 0,05). Um teste de aproveitamento foi feito em gémeos de sexos diferentes, ambos cursando a mesma série do primeiro grau. Usando os dados a seguir, teste a hipétese de que o desempenho médio é igual em meninos e meninas (« = 0,05). Pan de ‘serena: 1 ae Set eeu Set Onan 0) Menino: (60185) 170) 52) 657, 00; 63) 70) Menina: eto fe ee oy Verifique se as localidades a seguir diferem quanto & freatiéneia dos diferen- tes grupos sangiiineos do sistema ABO (a = 0,05). A B AB ° Total Cidade | 3 ° 1 a7 100 Cidade 20 w 8 45 100 Total 7 2 10 82 200 Uma moeda foi langada ao ar 200 vezes ¢ em 112 langamentos o resultado foi “cara”. Pode-se afirmar que esta moeda é honesta, para a = 0,05? Certo pesquisador determinou o grupo sangtiineo ABO de 140 pessoas, ob: tendo 71 individuos 0, 65 A, 3 Be 1 AB, e afirma que esta é uma amostra epresentativa da populagéo caucasside do Rio Grande do Sul no que se refere a tal sistema. Qual sua opinido sobre a afirmativa sabendo que, segun- - - 133. 134. 135, Bioestatistica 24 do Dornelles (1998), a freqiténcia dos diferentes tipos ABO em caucasdides desse Estado é: individuos 0: 45,1%; A: 41,3%; B: 10,3% e AB: 3,3%? Em um estudo que envolveu 47 alcoolistas gatichos do sexo masculino, fo- ram encontrados transtornos de depressao maior em 18 pessoas (Bau e cola- boradores, 2001). Estime a percentagem de individuos com esse tipo de trans- torno na populacao de onde se originou a amostra. A tabela a seguir mostra a classificagao de 511 recém-nascidos, estudados em dois hospitais de Porto Alegre entre janeiro de 1986 e fevereiro de 1987, quanto ao grupo étnico e ao tamanho em relacio a idade gestacional (Pedrol- lo, 1988). Entre parénteses estd a percentagem de criancas dos varios tama- nhos dentro do grupo étnico. Verifique se existe relacdo entre essas varidveis (a= 0,05). Tamanho para a ldade gestacional Grupo étnico Grande ‘Apropriado Pequeno Total Caucaséides 72 (23%) 233 (74%) 10 (3%) ‘315 (100%) Negréides 23 (12%) 159 (20%) 156%) 496 (100%) Total 5 381 2 Bit ‘Mufioz e colaboradores (1987) comparam 30 homens que tomavam chimar- Go diariamente (“casos”) com 30 homens que no tomavam mate ou usa- vam a bebida no maximo uma vez por semana (“controles”). Os grupos eram semelhantes quanto & idade, ao uso de alcool e ao fumo. Treze dos 30 casos apresentaram esofagite, enquanto 6 dos 30 controles apresentaram esta ca- racterfstica. Por outro lado, foram feitos diagnésticos de gastrite em 6 dos 30 casos e em 4 individuos controles. 135.1. Compare os dois grupos entre si, quanto a freqléncia de esofagite, usando uum teste unilateral (a=0,05), pois se esperaria que bebedores freqiientes de mate tenhiam maior risco de desenvolver a doenca devido a uma poss{- vel lesdo térmica 135.2. Realize a mesma comparagdo no que se refere A gastrite (também teste unilateral). 135.3. Galcule 0 tamanho da amostra necessario para demonstrat, com poder de 136. 80% e usando um teste unilateral com a=0,05, que a diferenca observada centre os grupos quanto ao niimero de casos de gastrite € real. (Observa- Go: para um teste unilateral, use 95 = 1,64) Em um estudo envolvendo tartarugas da espécie Phrynops hylarii, Foglia e colaboradores (1955) mediram a quantidade de glicogénio muscular (mg/ 100 g tecido) em individuos integros e naqueles cujo pancreas foi retirado. A média + DP nos animais integros foi 8404291 e nos pancreatoprivos fot 6094222. A diferenca entre essas médias nao foi estatisticamente significati- va (¢ = 1,93; P>0,10). Qual deve ser o tamanho da amostra para mostrar estatisticamente essa diferenca, se ela for real, com poder de 90% e nivel de significdncia de 0,052 | 22 Sidia M. Callegari-facques 137. A freqiiéncia da inverséo F do cromossomo IIL foi estudada em populacées urbanas de Drosophila willistoni. Os exemplares foram coletados em zonas consideradas de alta, média e baixa urbanizacdo, na cidade de Porto Alegre (Valente e colaboradores, 1993). Os dados foram comparados entre si e com um grupo-controle nZo-urbano por um teste qui-quadrado, obtendo-se o va. lor 2 = 70,39. Analise as tabelas a seguir, referentes aos dads obtidos e aos residuos caleulados apés o teste, e conclua sobre a relacio entre urbanizacéo : e freqiiéncia da inversio F em D. willistoni (a=0,01). ‘Nimero de individuos portadores da inversao F Urbanizagio Tita ntermediaria Baba Controles Total 3 aa eat 23 1349 5 1201 1502 658 3875 338 16x 72188 381 Ea 12% 25% 28% 25% 25% Residuos Urbanizacto lnversiof All Tnlernediavia Baba Controles Sin “Tae 017 488) -037 Nao. 789 0.17 38 037 138. Foi medida a largura da carapaca (mm) em 200 caranguejos da espécie He- patus pudibundus, do litoral do Estado de Sao Paulo (dados de RLM. Mante latto, Dep. Biologia-USR, Ribeirao Preto). Suponha que um bidlogo deseja usar estes dados para fazer as seguintes comparagées (apenas estas), apés analisé-los por meio de uma ANOVA: machos jovens x fémeas jovens, ma- chos adultos x fémeas adultas, e fémeas adultas x fémeas com ovos. Ele pretende usar, nas comparacGes, o procedimento de Bonferroni. As médias para cada grupo e parte dos resultados esto a seguir. Qual a conclusdo do bidlogo (a=0,05)? ANOVA: ‘Macho Macho Fémea Fémea Fémea Causas de Jovem _adufo Jovem adult cloves variagdo saat média 256 624 27,8 484 509 Entregrupos 16890 4 a TO) ch) ee re Total 31.456 139. A transferéncia de ninhos da tartaruga marinha Dermochelys coriacea é feita quando 05 ovos esto sob algum tipo de risco (inundacio, erosio da areia, predacio, transito excessivo de pessoas, etc.). Com 0 objetivo de estudar 0 efeito do manejo dos ninhos sobre a descendéncia dessa tartaruga, Morisso e Krause (2001) compararam o niimero de nascidos vivos/ninho e o nimero de natimortos/ninho em trés condigdes: (A) ninhos mantidos in situ, (B) ninhos transferidos para um cercado de incubacio e (C) ninhos transferidos 140. Bioestatistica 213 para a praia. Tanto o miimero de nascidos vivos quanto o de natimortos tém distribuigio ndo-gaussiana, por isso os autores utilizaram testes estatisticos ndo-paramétricos. Os resultados obtidos estdo a seguir. Qual foto teste utili. zado e qual a concluséo (@=0,05) para cada uma das variéveis? Varivel ‘insu ‘Transf. plcercado Transf. plprala Valor caleula~ (a= 113) (= 13) donoteste Nenascidos vives: media 62,7 a7 74 DP (28.1 23.7 fe nalimortos: média 3,0 19 04 oP 60 24 ‘Aplique um teste nfo-paramézrico aos seguintes dados, para justificar (ou no) a afirmativa de que as populagées indigenas de lingua Caribe so maio- res do que as de lingua Tupi (a=0,05). Caribes Tunis Tbs Neds Tribe Nido Indviduos Individuos 7 258 1 a 2 600 2 400 3 200 3 350 4 2000 4 344 5 2000 5 593 6 2000 6 710 7 2700 7 710 8 3000 8 2500 9 ‘4000 Fonte: Gallegari-Jacques e Salzano, 1989, Respostas dos exercicios ee en ges 10. i. 12. 13. 14. 15. 16. wv. 18. 19. 20. a1. 22, 2) 15 25 75115 8; 55 15 1 3) a) 13,9% b) 27,8% c) 50,0% d) 5,6% 4) a) 0,028 b) 0,306 d) 0,084 1) n = 6; Ex = 15; 3x2 = 55; média = 2,5 graus; mediana = 2,5 graus; var = 3,50 (graus)®; DP = 1,87 graus; 2) n = 7; Ex = 19; De = 71; média = 2,7 graus; mediana = 3 graus; var = 3,24 (graus)®; DP = 1,80 graus média 4,6; moda: 1 |~3; DP = 2,52 9; var = 10,5; s = 3,24 ,63. Amplitude: 32 nas duas. Desvio padrio. Série 2. CV idade = 0,76; CV glicemia = 0,10. A idade varia mais. Miimero de nascidos vivos: extremos = j14:3a= -25 e 131, Tempo de incuba- co: extremos: 46 e 94. A distribuigdo para 0 niimero de nascidos vivos nao &normal, pois ~25 é um valor impossivel no contexto; jé o tempo de ineuba- do pode ter uma distribuigéo normal. 0,472; 0,0668; 0,1587; 0,76975 0,1044 Oe 1,16; 0 € ~0,33; 1,96 € 2; -2,3 e2 . 0,9332; 0,0228; nao se desvia significativamente de 1 11,5%; 0,2853; 186,1 cm ou mais =1,28; nota = 5,7 0.843 74,2 175,5 -224,5. Esta média amostral desvia-se significativamente de 200 mg/dl. ‘hay = ~4,24. Amédia na populacao do sul da América do Sul é significativa- mente menor. A curva de distribuigao amostral de médias (DAM) é sempre mais estreita que a distribuicdo de.x. A DAM para n = 16 ¢ mais estreita que a DAM para n= 4, pois EP(n = 16) ¢ menor do que EP(n = 4) Zax = ~3,125. No. O mais provavel é que esta amostra provenha de uma populacio com media menor, pois 0 desvio entre 1,3 ¢ 1,46 estatisticamente significativo. Zaye = ~17,32. A retirada da hipéfise estd associada a uma reducdo na glicemia. Média = 7,5; DP = 1,08; te = 3,675 > toos,¢ = 2:447. A nova variedade tem sementes maiores. fue = ~1,787. Néo ha diferenca estatisticamente significativa entre as mé- dias. 0 nivel de chumbo no solo nao foi afetado pela queimada. Média = 14; s = 2; faye = -3,164; fyos.9 = 2,262. As mulheres apresentam nivel de hemoglobina mais baixo. 332. 31, 32. 33, 34, 35, 36, 37, 38, 39, 40. 41. 42. 43. Bioestatistica 215 fone 11,900 > tooy.7 = 3,499. Em genitores de criancas com fenilcetontiria, a concentracéo de fenilalanina é mais alta do que na populacgo em geral EP = 0,2064. IC yoy? 4,25 ~ 4,95; IC yyy? 4,18 ~ 5,02. 0 1Coas,€ mais preciso, ‘mas a confianga é menor. fate = 3,014 > fog = 2,306. 0 conterido de material slido no rio Raumentou. 5. EP = 0,616. Cys, 13,96 ~ 16,44 ppm. 1) IC ogy: 252,4~ 311,65 2) NAO, se Fosse gaussiana, a média seria um valor situado mais ao centro da distribuicéo. 44,61 - 40 = 25/N61, entaos = 18 F = 1,38, ndo-significativo; 1, = 3,63. Pacientes com proteintria tém glice- mia mais alta, Comprimento: F = 1,14, ndo-significativo; fy. = 1,29; mandibula: F = 1,47, no significativo; (yc = 1,99. Ndo existe diferenca significativa entre localid des quanto ao comprimento do corpo e largura da mandibula, ento parece ndo existr relacio entre niimero cromoss6mico e estas medidas morfolégicas. F = 1,00, nao-significativo; t= ~ 4,00; tog = 3,355. Os usuarios sen- tem-se mais confortéveis usando a pipeta do tipo B. Comprimento: F = 1,4, néo-significativo; taj. = -0,92; diametro: F = 2,0, nido-significativo; ¢,4, = 19,9. Nao existe diferenca entre locais quanto 20 comprimento, mas 6 didmetro dos ovos em Sapiranga é maior. MHtb: F = 2,3, significativo; t,, = 1,88, gl = 25; 0,05 foasis = 2,306. A nova dieta determina um aumento no peso corporal das cobaias. toa = 2,97 > tos untueei9 = 1.833. O novo medicamento reduz a pressio arterial sistlica ~ PAS (aqui se justifica um teste unilateral, porque a pergun- ta era: “Hé ou nao uma redugiio na PAS?”). fone = 4,02 > tgoy.24 = 2,797. 0 medicamento M teduz o colesterol plasmé- tico (aqui o teste é bilateral porque o pesquisador queria identificar um efei- to, fosse ele de aumentar ou diminuir o colesterol) 7 = 0,94; taic = 4,77 > loos = 3,182. Existe correlacio positiva muito forte entre 0s resultados dos dois instrumentos. foie = ~ 4,655 fate = ~ 2,67; lara = 2,763. Existe correlacio negativa forte entre nivel de alfa-fetoproteina ¢ idade gestacional, mas no hé correlagio entre niveis de alfa-fetoproteina e de fibronectina, para o nivel 0,01 de significéncia. Existe correlacao inversa forte entre VT e temperatura (P < 0,01) Ex = 72; Ux? = 636; By = 12,6; Dy? = 19,28; Soy = 110,65 J = 0,096 + 0,163x; EP = 0,010; fj, =16,14. Em criancas de 4 a 12 anos, a capacidade pulmonar depende da idade. Para cada ano a mais na idade, espera-se um aumento médio de 0,163 Ina capacidade pulmonar. Yx= 15; De = 55; By = 136; Sy? = 3222; Day = 292. f = 29,52-2,74%, EP = 0,324; t,4= ~ 8,456. Para cada unidade adicional de calcério adiciona- do 20 solo, espera-se uma reducao de 2,74 na percentagem de anomalias das células germinativas. 1) Existe correlacdo positiva forte entre In (largura do abdémen) e In (lar- gura da carapaca). | 2G Sidia M. Callegari-facques 45. 47. 49. 50. 52, 53, 54, 55. 56. 57. 58. 59. 60. el. 62. 63, 64. 65, 66. 46. 2) t= (b-0)/EP = 1,25/0,15 = 8,33 (P<0,001). Espera-se um aumento de 1,25 unidades de In(larg abdémen) para um aumento de uma unida- de de In(larg carapaca). (b - D/P = (1,25 - 10/0,15 = 1,67 < togsay = 2,093. Ndo hd evidéncias suficientes para rejeitar a hipétese de isometria. No gréfico de colunas, a linha horizontal nao é um eixo e as colunas no sto justapostas, como no histograma. 0,4x0,4 = 0,16 0,5 (0,967) = 0,4835 1) 0,873; 2)(0,03x0,9) + (0,97x0,1) =0,124 3)0,003 9/16. Nao, pois a probabilidade “nao tem meméria”. 3) 1) (0,32+0,08)$ = 0,064; 2) (0,4x0,6) + (0,6%0,4) + 0,47 = 0,64 1) 0,478; 2) aprox. zero; 3) 0,372; 4) 0,372+0,478=0,850; 5) 0,026 Binomial: 0,075 +0,035+0,012+-0,003=0,125; normal: z = 1,37, logo area = 0,085; normal com correcio: = = 1,14, logo drea = 0,127. Distribuicéo normal com corregio produz probabilidade mais proxima da obtida pela bi- nomial. 7.7% ~21,5% 11,8% — 52,2%6, A pouea preciso é devida ao tamanho amostral. Para au- rmentar a preciso € necessério estudar um nimero maior de pacientes. ‘rau, = 1,83 < fogs = 1,96. Nao houve mudanca de comportamento. China: gy = 1,385 Ind: 2,4, = 8,01. A prevaléncia de esofagite no Rio Gran dedo ‘Sulé semelhante & da China e menor do que a do Ir = 1,53 0 7? ae = 2,34 (P>0,10). Nao, pessoas com niveis de colesterol vado sao igualitente freqientes nos dois géneros dessa popula Fue = 3,26 01 2ay. = 10,61. O mosaicismo eromossémico é mais freqtiente na cultivar CNT-10. Peeas = 137.4 > 2 coors = 26,12. A distribuigdd da ovoposigio nio é uni forme ao longo dos meses de margo e novembro. P ca = 23,7 > 2 cous = 15,09. Aos sdbados ocorrem mais acidentes de trabalho, nesta empresa Peue = 7.11 < Poosa = 9,49. Nao. As diferencas entre os ntimeros observa: dos € os esperados de acordo com Nilsson-Ehle séo causais. P ete = 10,96 04 Ze. = 3,31. Entre as pessoas com problemas de tiredide, a proporgio de pessoas sensiveis ao PTC é menor do que na populaggo em geral. #eeac = 1.60 < ops = 5,99. Néo existe associacko entre genero e cor do pelo nestes roedores. Infcio da vida sexual: 7? = 7,97; resduos para “Casos”: +2,02; -0,12; -2,48. DOQBI"03: 2? gong = 13,84. Mulheres com cancer de cérvice uterina inicia- ram mais cedo a vida sexual. Nestas mulheres, a freqiténcia do alelo DQB1°03 6 maior do que no grupo controle. F cag = 93,8. Residuos para “Cancer de pulmo": 5,94; ~4,07; +1,05; +6,74; 3,57. O risco de cancer de pulmao aumenta com a quantidade de cigarros fumados diariamente. Idade: 7... = 4,62; contaminacéo racial aparente: 7? = 1,22. A freqiién: cia de cromossomos acrocéntricos (tipicos do gado zebuino) é menor em touros mais jovens, logo a mistura com zebu esta diminuindo no plantel. Por Bivestatistica 27 outro lado, a varidvel “mistura racial aparente” no é um bom indicador de miscigenaco, pois nao foi encontrada associaco entre as varias categorias desta varidvel e a freqiténcia de acrocénticos. 67. 7? cae = 0,83. Os tipos Hp nao tém freqiiéncias diferentes nas duas formas de ‘esquistossomose. 68.1) Ser maior; 2) Ser menor, pois a variacio entre individuos altos é menor do que a observada na populacao como um todo. 69. 35 em cada amostra, no minimo. 70. No minimo 99 individuos. 71. $QEnere = 17,8; SQ Dentro = 6,9; QM Entre = 5,93; QM Dentro = 0,86; F = 6.95 dea 5,355 5,61; 4,40; 1,75. O ponto B apresenta dano maior do que os pontos A e C, que néio diferem. Os resultados relativos ao ponto D sio incon- clusivos. 72. Fee = 91,74 (P<0,01). Tukey: dage= 16,215 1,505 12,485 4 oo5:.52 = 3,399. Os heterozigotos I e V diferem dos controles, mas nao diferem entre si, logo, © objetivo nao foi aleancado. 73. ee = 5:72; 0,28; nao testar; 5,61; nfo testar; 5,45. EP = 0,43754 oos.4;8 = 3,845. A lagoa Inferndo apresenta as menores concentracées de silica; 05 ‘mananciais restantes néo diferem entre si 74. SQDieta = 96,4; SQ Peso =113,3; SQ Restduo = 54,3; QM Dieta = 48,2; QM Peso = 28,3; QM Reslduo= 6,8. dietas = 7,105 dey. = 5,325 2,405 2,92; F pes 4,17. Aredugio de peso é maior na Dieta 1 do que na 2. Os resultados referentes a Dieta 3 sao inconclusivos. 75. Procedimentos: gl = 3; F = 0,50 (P>0,05). Gestantes: gl = 6; F = 2,45 (P>0,05). Nao ha diferenca estatisticamente significante entre procedimen- tos. Também nao hé diferenca significativa entre gestantes. 76. SQ Entre = 40,40; 8Q Dentro = 22,74; QM Entre = 20,20; QM Dentr 1,52; Faye = 13,3. Gea: 7,305 3,38; 3,92. doos.r5;3 (Tukey): 3,674. O nivel de hemoglobina nao difere em pessoas com HB 8/beta talassemia e HB SC; em pessoas com fenstipo HB S, o nivel de hemoglobina é menor. Hb S: Hb Siosta-talassemia Hb SC esta Be 70 12, a 6 6 6 77. Faye = 7,85 (P<0,01). SNK: doy = 5,41; 1,283 3,835 Gog, y14 = 3.8205 3,113; 3,113, Criangas que receberam BGG por vie oral ou intradérmica apresen- tam resposta de igual intensidade, tendo o diémetro da reacao maior do que das criancas que nao foram vacinadas. 78. Us = 51,5. Nao hé diferenca entre machos e fémeas. 79. Tee = 8,0. A freqiiéncia de anomalias de fuso é mais alta em individuos expostos ao chumbo (P = 0,05). 80. A,=180; A, = 180,5; 2d? =383,5; r, = -0,064, néo-significativo. 81. Se o nivel de oxigénio tem distribuicdo normal, usar t’, pois F foi significati- vo. Se esta varidvel nao tem distribuico normal, ou néo se deseja usar um teste paramétrico, usar o teste de Wilcoxon-Mann-Whitney ou o Kruskal- ‘Wallis para k = 2. Com todos os testes, P < 0,001 e conclui-se que o nivel de oxigénio é mais alto na lagoa Jacaré. 2B Sida M. Callegar-facques 82, 83, 84. 8s, 86, 87. # mexemar = 12,90. O medicamento reduz significativamente a dor nas pri- ‘meiras 24 horas apés a aplicacdo. Pr(1)= 0,2937. Como este valor jé é maior do que 0,05, néo é necessério realizar os demais cdleulos. Conclui-se, pelo Teste Exato de Fisher, que nao hé diferenca significativa na freqiiéncia da retinopatia de base em sobrevi- ventes e nao-sobreviventes. (1) H= 2,18; P > 0,20, (2) H= 79,10 (P < 0,001); EP= 4,69; Q critico= 2,639, ‘Tupi (freqiiéncias mais altas) difere dos demais, que no diferem entre si. H,= 13,87; gl = 3; P<0,01. EP= 4,03; Q= 3,40; 2,91; 2,665 0,74; néo se testam os demais. O rio Mogi-Guacu tem niveis de nitrato mais altos do que as lagoas, que nao diferem entre si; os resultados para 0 cérrego Cafundé sio inconclusivos. 17, = 0,638; se nao for feita correcao para empates, entio rs = 0,643. “EE.Risher: P = 0,0046. Com 0 tratamento A ocorrem menos complicagdes. EXERCICIOS DE REVISAO 99, 100. 101. 102. 103. ¢, 104, 105 107. 108. 1) Média = 11,9; mediana 125 DP = 1,545 2) IC yg 11,0 -12, ). 1) 0,5; 2) 2, = -0,8; 2, = 1,6; prob = 0,7333; 3) 0,0228 HP = 0,188. 0 ICyyy £8,287 ~ 9,073, |. Média = 12,1; mediana = 11; DP = 5,28 1) s entre -0,84 e 0,84; 2) « entre ~« e -0,61; 3) z entre -0,52e +00 |. Média = 1,92; md = 1,50; s = 1,04. A mediana é a melhor medida de ten- déncia Distribuic conforme n. Todas as propriedades relativas ds reas da curva normal so vélidas. >. 1) 0,4641; 2) 0,0968; 3) 0,88495 4) 0,0183; 5) 0,7213 5. Média = 65,1; md = 65; moda = 65; a = 16; s? = 19,66; s = 4,43 1) . Mediana (SC) = 5; mediana (HO) = ~0,33, 63%; 2) = = -0,84, 7,9 m; 3) = = 1,28; 16,8 m ou mais, (© mimero de consultas parece ser maior nas gestantes do Hospital de Clinicas (para confirmar esta concluséo, seria necessdrio realizar um teste estatfstico ndo-paramétrico, por exemplo, 6 teste de Wileoxon-Mann-Whitney) 1) 0,0475; 2) 0,9332 Média = 0,88; DP = 0,11; IC gg 0,81 - 0,95 ,98 > Boos yay = 1,645. A pressdo arterial sistdlica é mais elevada em Jovens que ingerem diariamente muito sal se eles pertencem a familias nas quais hd pessoas com hipertensao essencial. EP = 2, Aregifo de ndo-significdncia fica entre 46,08 ¢ 53,92 tate = |-2,41| >to 95,49 = 2,021. Rejeita-se H, e conclui-se que a capacidade vital esté diminuida nesses operdrios. Zig = 2,08 > 1,96. Ha diferenca significativa na envergadura das asas dos passaros dos dois locais; logo, existe variacio geografica. . 2 = ~0,40; drea = 0,3446. O desbaste atingird 34,5% das drvores. 106. Média = 3092; s = 301,55 t.a. = - 4,06; 0,001 to,05;39 = 2,042. A deficiéncia em vitamina K esté associada a niveis mais baixos de protrombina. Pressio arterial sistélica: F = 1,0 (nao significante); t,4. = 4,77, P<0,001, Mulheres apresentam niveis de pressio arterial sistélica diastolica superio- res a0s dos homens, mas néo diferem destes quanto a glicemia e quanto a0 indice de massa corporal. 109. 110, 11. 112, 113. 14 1s. 116. uz. 118. 119. 120. 121 122, 128 124. 125. 126. 127, 128, 129, Bioestatisica M9 Média = 6; s = 4,35; t,4, = 4.37; 0,01>P>0,001. Exercicio moderado (600 kgm/min durante 5'min) aumenta a freqiiéncia respiratéria, mesmo em individuos treinados. 1 = 0,67; fy. = 1,80; 0,20>P>0,10. O coeficiente de correlagio nao é signi- ficativo. Nao foi possivel demonstrar que existe correlago entre aptido produtividade. Existe correlagio regular entre cénjuges no que se refere a altura e ao peso, fraca para comprimento do nariz e nao hé correlago quanto & espessura dos labios. Os dados indicam, portanto, que as pessoas em geral procuram cs ges relativamente parecidos com elas mesmas quanto ao peso e & estatura; também o fazem, mas em menor grat, quanto a0 comprimento do nariz; mas 0 mesmo néo ocorre quanto & espessura dos labios. 1.3) j= -9,06 + 0,28x; 3) EP = 0,12; taje = 2,33. NAO hd evidéncias sufi- cientes para afitmar que existe relagéo entre peso e comprimento rostro- anal. 4) Com uma amostra maior, pode-se afirmar, com pequena probabili- dade de erro (P<0,001), que 0 peso depende do CRA, esperando-se um au- mento de 0,30 g para cada aumento de 1 mm no comprimento rostro-anal, 5) A taxa de crescimento diminui (8 razdo de 0,20g/mm) com o aumento do comprimento rostro-anal F = 2,19;0,10>P>0,05, logo as variancias nao diferem. t,y. = 5,83; P<0,001, © comprimento do crénio é maior nos animais de Tijuca, F = 1,00; P>0,10, logo as variancias no diferem. t,. = ~0,33; P>0,20. Nao hé diferenca entre métodos de ensino. F = 9,00; P<0,01, logo variancias diferem. t’,. = 3,641; gl’ = 26; 0,01>P>0,001. A atividade de renina plasmética esta aumentada em indivi. duos diabéticos. 1) 0,47%0,83 = 0,390; 2) 0,47x0,17x0,53x0,83= 0,035; 3) (0,47x0,83)2x0,53*0,83= 0,067 1) 0,004; 2)0,187; 3)0,138+0,037+0,004= 1) 0,590; 2) 0,74 1) 0,026; 2}0,026+0,006+0,001= 0,033; 3) 0,268+0,107= 0,375 Fate = 6,52 OU 2 cong = 42,35 (gl = 1); P<0,001. HA mais fumantes na Santa Casa, Irate = ~1,99 01 7? cong = 3,97 (gl = 1)5 0,05>P>0,02. A nova variedade & mais resistente. F cae = 8,85; 0,02>P>0,01. Nao sio compativeis, hd mais individuos com 3 ‘ou mais acidentes cae = 1,87; P>0,20. Nao ha diferenca entre regiGes. TC Sou? 0,191 — 0,245 Zee = 5,61 OU 7 ng = 31,46 (Bl = 1); P<0,001. Nos alcoolistas que comegam com problemas mais cedo, a freqiiéncia de alucinagdes (72%) é maior do que naqueles nos quais os problemas devidos 20 dleool comecam mais tarde (21%). # cate = 3,82; 0,20>P>0,10. Nao hé efeito do gradiente de luminosidade sobre a distribuicao da graminea. Fate = 419 OU 2° cong = 17,58 (Ql = 1); P<0,001. A droga reduz a proporcao de pessoas que tém Vomitos pés-operatérios. Feu = 2,04 OU 2 conig = 4417 (Gl = 1); 0,05>P>0,02. A proporgio de pes- soas favordveis é maior na cidade A. Trata-se da comparacio entre duas amostras pareadas. Média = 1;s = 1,927; fae = 1,47; 0,20>P>0,10. Nao existe diferenca entre meninos e meninas quanto 20 aproveitamento escolar. 179 220 Sidia M. Callegari-Jucques 130. 7 cae = 11,63; 0,010>P>0,001. Existe diferenca significativa entre locais. A fnilise de residuos mostra que as diferencas sao devidas principalmente ao ‘grupo A (cidade I em mais individuos do que o esperado) e AB (cidade I tem ‘menos pessoas do que o esperado). Analise de residuos (os marcados com * so significativos ao nivel 0,05): A B 5 ° Cidade | 2,08" 1,88 2.80" 0.28 Cidade 2.08" 1,68 260° 028 131. Sie = 1,62 00 2 conig = 2,64 (gl = 195 0,20>P>0,10, Nao hé evidencias de qué a moeda seje desonesta. 132. 72 gue = 13,73; 0,01>P>0,001. A amostra nao é representativa da popula- {40 gaticha, devido principalmente a falta de individuos B. 133. P = 0,383; EP = 0,0709. A proporgao verdadeira de homens alcoolistas gaui- cchos que apresentam depressio maior é um valor entre 0,233 e 0,533, com 95% de confianca. 134. # cae = 13,695 0,01>P>0,001. Existe associacio entre grupo étnico e tama- ho do recém-nascido. Criangas grandes para a idade sao mais freqiientes em caucasdides e pequenas sio mais freqiientes em negrdides. Andlise de residuos (0s marcados com * so sigaiictivos 20 nivel 0,05): Grande ‘Apropriado Pequene Caucaséides ae “72 228" Negroides 16 472 228" 135. 1) Esofagite: 2,4. = 1,67; 0,05> Pyyuarret 20,025. Existe diferenca entre grupos. A esofagite é mais comum em homens que tomam chimarrao com maior freqtiéncia, 2) Gastrite: 4 = 0,35; Pysigerat 70-20. Nao foi encontrada diferenca signi- ficativa entre grupos quanto a freqiéncia de gastrite. 3) P, = 0,2; P, = 0,133; P, = 0,167; diferenca = 0,2 - 0,133 = 0,067; Foosuniert = 15643 W = Zo 20.ntert = 0,845 NO Minimo 380 pessoas em cada grup 136. No minimo 28 individuos em cada grupo. 137. 7 ae = 70,395 P < 0,001. A freqiiéncia da inversio F esta inversamente associada ao grau de urbanizacao: ela é mais freqiiente em zonas de baixa urbanizacéo (29%) e menos freqiiente (12%) nas areas de alta urbanizacao. Nas zonas com grau intermedidrio de urbanizacio e nos controles, a inver- so F ocorre na freqiiéncia esperada ao acaso (26%). 138. QM Entre = 4222,5; QM Res{duo = 74,7; F = 56,5. dpoqs = 0,017, toomiss = 2,358. Existe diferenca significativa entre sexos nos adUultOS (yong = 2.772) ‘mas nao nas formas jovens (yy, = 0,802). Também nao ha diferenca estatis- ticamente significativa entre femeas adultas com e set OVOS (tq; = 1,035). 139, Teste de Kruskal-Wallis. Nascidos vivos: gl = 2, P<0,001. HE diferenca entre ‘manejos quanto ao niimero de nascidos vivos observado por ninho. Natimor- 2, P>0,20. O mimero de natimortos nao muda com o manejo. 2, P<0,05. As populaces de lingua Caribe séo maiores do que as de Mngua Tupi. 140. TABELAS Tabelas 223 TABELA At Distrbuigto normal: valores de 2 areas entre a média (zero) @ 2 z Weaenie|z dreaente]2 sreacne]z aroaente| 2 treaenve]z en eane ez ez Qez ez Doz ez 0,00 0,0000 02123 [a2 0.3686 oss [224 04875 [280 0.4074 0.01 o,0040 02187 | 413 03708 04565 |225 04878 0.4975 0.02 0,080 02180 | 114 03729 04554 | 226 0/4881 0.4976 0.03 0.0120 oz | 4115 0.3740 os64 | 227 0.4004 oa77 0,04 0,0160 02957 | 4,18 0.370 04873 | 228 04087 0.4977 0.05 0.0199 oza01 | 4117 0.3790 04582 |229 o,4890 04978 0,06 0.0298 02924 | 4:18 0.3810 04591 | 230 o4e93 0.4979 7 ooze 02357 | 4,19 0/3830 oasse | 231 04896 0.4978 008 oos19 0.2389 | 1120 0.3849 0.4608 04898 | 2188 04980 009 0.0359 02422 | 121 03869 | 1.7 0.4616 ost | 289 ol4aet 0.10 o,0308 0.2454 | 122 ogee |1.78 0.4625 0.4904 | 290 04981 ot 0438 02486 |1.23 oge07 | 179 0.4633, 0.4906 |201 o.4982 oe o,0478 02517 |124 0.3025 [1180 oaest 04909 | 292 0.4082 043 00517 02549 |125 0.3844 [1181 04640 Daa |283 04083 014 0.0557 02580 | 126 o.s062 | 1182 0.4856 04913 |2.04 0,<984 015 0.0508, 2611 | 127 0.3980 |1183 04664 ose |2.95 0.4984 016 0.0836 02662 | 428 0.3907 | 114 oeri o4a18 | 2.96 0.4085 07 0.0875 02673 429 oo1s | 1185 ose7e 04920 |2.97 0.4985 018 aorta 02704 }1'30 04032 |1.86 0.4686 oa9z2 2.98 0.4086 0,19 o.0753, o27s4 | 131 40d | 187 0.4693 04925 |2.09 0.4086 020 00703 o2764 |1s2 04088 |7.88 0.4699 04927 | 3,00 o.4ge7 021 0.0832 2794 | 1/33 o4os2 | 189 0.4708 0.4923 | 3.01 0.4007 ooa71 }07e 2823 |134 04089 | x90 o47i3 04931 |3.02 0.4987 0310 |079 oze52 |135 oats |e O79 04932 |3.03 0.4988 cose |oe0 o2ee1 |136 o4is1 | 192 04726 oes |304 0.4988 0987 Joi ozo |1is7 o4ia7 [1193 04732, 04938 | 3,05 0.4980 a1026 |oe2 o2e39 |1/38 oete2 | i194 4738 04938 | 3.06 0.4089 0,106 [0.83 0.2967 |1139 oi77 | i195 0.4744 04940 | 3.07 0,4089 ot0s |oe4 02995 |1140 04192 | 1198 o.47s0 o4g4t |3,08 0.4900 ota [oes oes | x41 0.4207 | 1197 0.4756 04943 | 3:09 0.4000 0.1178 | 0,88 0.3051 | 1.42 0.4761 0.4945 014980 oxai7 Jos? 03078 | 4.43 0.4767 0.4946 0491 0.1255 ]088 03106 | 1.44 ort 04948 0.4991 0.1293 |os ostss | 145 oar7s | 257 04049 0.4391 oat |o90 osts9 | 1146 o47e3 | 258 04951 0.4992 0.1368 Jost ostes | 1147 0478 [259 0.4052 0.4992 0.1408 Jose 0.3212 | 1148 04793 | 2160 0.4053, 0.4992 0.1443 ogaa8 | 149 04798 | 261 0.4055 902 0.1480 0.3264 | 150 262 0.4958 0.4993 01817 013280 | 451 263 04957 0.4303 0.1554 oats | 152 264 0.4959 0.4903 0.1891 03340 | 4163 285 0.4960 0.4903 oeze Joss 03065 | 1/54 268 04961 0.4904 01664 }099 0.3389 | 1155, 287 0/4062 0.4994 0.1700 | 1.00 o.se13 | 156 268 0.4963, 0.4994 01736 | tot osase | x57 289 04964 0.4994 046 ot7re | 102 ost | 1/58 270 04965, 0.4995 047 0.1808 | 1.03 0.3485 | 1159 271 o4066 | 3:35 04096 048 01844 | 1108 0/3508 | 1160 272 o4s67 |340 0.4997 o4s 0.1879 [1405 013531 | 1'61 273 oes |3.45 04907 080 01015 | 1105 013554 | 4.62 274 0.4969 |3,50 0.4998 ost oss | 407 0.3577 | 1.63 275 o4s7o |3.60 0.4098 os2 0.1985 | 1108 0/3599 | 1,64 2768 aor |3,70 0.4999 59 2019 | 4109 03621 | 1165 277 04972 | 3,80 0.4999 ose 02054 | 1:10 03643 | 1,05 278 04973 |3,90 049995 oss ozo [111 ose6s [167 04825 | 223, 279 oa7a_| 400 _0,49997 224 Tabelas |A2 Valores erltons da distibuigéo ¢ de Student er a CT) at Unilateral: 0,20 010 __—0,05——0,025—ot_—0.005 0.0005, 4 1376 «30786314 12,708 «81,821 «63,686 636,578 2 jst 1886 «-2,920 «4.303 6.965 © 9.925 31,600 3 og7s 1638 «2.353 3.182.5a1 Saat 12,924 4 oot tsa 2.492 2.778 «3.747 4.804 8.810 5 og20 14762015) 287t 3.965 «4.082 6.869 6 090s 4440 4.08827) 3,43 3,707 8,958 7 0898 1415118952985 2.988 «3.499 5,408 a ogo 1.307 t1ac0 «2.908 «2896 3,355 5,041 a ges 7.983 11833 «2.262 2821 3.280 4,781 10 ogre 1372 1812-2228 74 3,889 4,587 " oars 4,863 1798 2,201 2718 3,108 4.437 2 \ og7s 1388 172 2.179 «2881 3.055 4.318 18 og70 ©1350 «4771 -2.460 «2,850 3,012, 4,221 14 ess 1351761 2.145 2824-2977 4.140 6 ges = 1adt 47832131 2,602 «2.947 4.073 16 096s 1337 4,748 «2,120 -2,683 2.921 4.015 ‘7 0963 1333 «1,740 «N10 «2.567 2,808 (3,965 18 oesz 1.990 4734 = «2t01= 2552-2878 3.922 19 est 4328 «1729-2003 2839-2881 3,883 20 oso 19251725 -2,088 2828 2.845 (3,850 at aso 49291212080 2818 28st 3,819 Py gs 4921171722074 28082813 8,782 23 gs 1319 «1714 2,069 «2.800 2,807 3,708 a 0957 13 t7it 2064 «2492 2,797 3.745 25 0gss 1.318 1,708 2.060 2485 «2,787 3.725 26 oss 131547082058 279 «2,779 3.707 ar oess 1.314 4,703, 205224792771 8.889 8 0855 1,313 4,701.08 «2467 -2,769 «3,674 29 Dasd 4311 1699-22085) 2462 2,755 3,660 30 cass 4,310 1697-23002 2.487 © 2,760 3,848 40 ost 1,303 t.684 «2021 2.423 «2.704 3,551 60 os43 1,296 «= t1e7t 2,000 «2,980 2,860 3,460 120 sss 128014658 1,980 2,958 2617 3.873 Infirto oes 4.282 11645 1.960 2.826.876 3,200 Tabelas 205 TABELA A3.1_Distibuiggo F: valores critcos para um teste bilateral («= 0,05) ‘aro ‘Gide Nomorador Denominador 128-4 =S OC 1 643799 —<864~—800—622—«837~—«AN_ «GBT «08S GD TS —GTT 2 38.51 3900 39,17 39,25 39,20 39.37 99,39 39,40 941 39.41 3 1744 1604 1544 15,10 1454 1447 1402 1437 14334 4 1222 10.65 898 890 884 aye 375 5 10.01 8,43 676 668 562 G57 652 8 aa 726 560 852 545 541 537 7 gor 654 490 482 475 471 487 8 78T 6.08 443° 438 430 424 420 ie eee or 410 403 398 391 387 10694 5.46 395 385 378 3.72 366 362 M672 526 376 368 359 353 347 343 2 855 5.10 361 351 344 337 352 328 13 gat 497 348 339 331 325 320 315 % 630 4.86 338 329 321 318 309 305 18 620 477 320 320 312 308 301 2.96 18 612 4.69 322 312 305 299 293 269 17 604 4.62 316 305 2.98 292 267 262 18598 4.58 340 301 293 287 281 277 19 592 451 305 298 288 282 276 272 2 587 4.48 Bor 291 294 277 272 2.68 21-583 442 982 348 325 309 297 287 200 2.73 268 2 579 438 37a 293 2a 2.76 270 265 2% «575 4.35 290 281 273 267 262 4. 572 432 287 278 270 264 259 25-569 420 295 275 268 261 256 2% © 588 4.27 2e2 273 265 2.59 254 2 863 424 2e0 271 263 257 251 2B 861 422 278 26a 261 285 249 2 © 859 420 276 267 259 253 248 30557 4.18 275 265 257 251 246 32553 4.15 271 262 34 5,50 4,42 2e9 259 38 «8474.08 286 257 38-848 407 264 2.55 4-842 405, 2e2 253, 42 840 403, 261 251 44-539 402 259 2:50 4% © 537 400, 258248 4% 535 399 256 247 60 «834 387 258 2.48 55 531 395 253 243 6 ©6523 393, 2a 241 65 528 391 249 2'39 7 525-389 431 297 275 259 247 238 8 522,388 3,28 «295 273 2E7 245 2.35 100-518-383 325 292 270 254 242 232 125 © 515 380 322 289 267 251 239 230 160-513 378 320 287 265 249 237 228 200 «810 376 318 285 263 247 235 2.25 400 «08. -3.72 315 282 2,60 244 232 2.22 1000 504 370 313 280 288 242 230 220 Infnto 5.02369 312 279 267 241 220 219 226 Tabelas TABELA A3.1 Distribuigdo F: valores erticos para um teste bilteal (2 08) (continuseo) giao ‘gi do Nomerador Denominador 747620 CA«S CTS 100-200 500 _infillo 7 96396709397 1001 7008 1008 1011 1019 1016 1017 1018 2 3943. 30,44 39,45 3940 3946 39,47 39,48 3048 99.49 99,49 39,50 99,50 3 1428 1423 14.17 1412 1408 1404 1401 13,97 13.96 1393 19,91 19.90 4 BBB «863 855 851 Bas BAT 832 829 827 8,28 5 646 640633 6.28 608 605 803 6.02 6 = 30 S24 8:17 5.12 492 488 486 4.85 7 480 454 447 aad 421 418 4184.14 8 413,408 400 3.95 374° 370 368 387 9 380 «374 367 361 340 337 335 333, yo 355 350 342 3.37 315 312 309 3.08, 1 338 380 323 212 309 303 2,98 298 292 290 288 2 321 318 307 295 291 287 282 280 2.76 274 272 13-308 303 295 2ea 278 2.7% 270 267 263 261 260 14 288-292 284 273 267 286 250 258 263 280 249 18 -2e9- 284 276 2e4 259 255 250 247 244 241 240 16 2e2 276 2168 257 251 247 242 240 236 233 232 17 275 270 262 250 244 241 295 233 229 226 225 18 270 284 2.56 2a 238 235 280 227 223 220 219 18 265 2.59 251 239 233 230 224 222 218 218 213 2 © 280-285 248 238 220 225 220 217 213 210 209 2 «2868-281 242 237 231 225 221 218 243 209 205 204 2 283 -2a7 239 233 227 221 217 212 203 205 202 200 2 250-244 236 230 224 218 214 208 206 201 199 197 2 BAT Dat 233 227 221 2:18 205 202 198 195 194 2 244-238 230 224 218 212 202 200 195 192 191 2% 242 218 209 200 197 192 1190 1188 org az 213 207 tor 194 1190 187 a5 2 237 21 205 195 192 188 185 1.83 2236 209 2.03 193 1,90 1186 1.83481 30-234 207 2.01 voi 188 184 1811.79 32 «281-225 218 210 204 198 1,99 1,88 1,85 180 177 1,75 34-228 222 213. Dor Zot 198 190 1185 182 177 174 1:72 38 «225 220 21 205 too 192 18s 1182 1,79 174171188 38-223 «217 209 203 198 1190 185 178 176 171 168 166 40-221 215 207 201 1194 198 183 177 174 169 166 164 4 =: 220-214-205 199 1192 186 181 175 172 187 164 162 4-248 212 203 1.97 191 184 180 1173 1:70 165 162 1,60 4 DAT 2.41 202 1,98 1189 182 178 1,72 1.09 469 1.60 1,58 43-218 209 201 194 1188 181 177 1,70 1.87 162 1:58 1,58 50-214 208 1,99 193 187 180 175 169 1.66 552i 205 197 190 184 177 1:72 1,88 162 6 209-203 184 188 482 174 «1:70 1.89 1160 6 207-201 193 186 80 1172 1188 181 1.58 7 206-200 181 185 17a 171 186 159 1:56 8 = 2081.87 1188 1a2 175 1188 183 8 153 300 200 1194 1185 1,78 471 164 198 152 148 425 197 191 N82 175 188 161 188 149 145 150 195 189 1180 174 «167 1159 154 148 142 2000193 187 178 171 164 158 Tet 144 1138 400 1190 1186 1174 68 160 152 147 120 1136 tooo 1881182 172 1165 158 150 145 1136 1132 infito 1487 1.80171 e487 14814334130 Tabelas 227 TABELA A3.2 Distibulcso F: valores crtloos para um teste bilateral (a= 0,01) giao ‘Gide Numerador Denominador 23 4 8 6 7 8 8 1 16212 19897 21614 22501 29086 23440 29715 29024 24091 24200 D434 —DaAZT 2 198.5 1990 ‘199.2 199.2 1993 190.3 199.4 1094 1994 199.4 1904 190.4 3 56,55 49,80 47.47 46,20 45,50 44,04 4443 44,13 43,08 43,68 43.82 43.50 4 3183 2628 24.25 23:15, 2198 21,62 2138 21.14 2097 2082 20,70 S 2278 1831 1653 158.56 4451 1420 13190 13:77 1362 i340 1328 6 1883 1454 12.92 12.09 1107 10,79 10.57 10,39 10,03 7 1824 1240 10.88 10.05 916 869 8.68 8.61 818 8 1483 14.04 960 8.81 798 789 7,50 7.34 701 8 1381 1011 872 7.96 713 688 669 6.54 623 10 12.83 943 808 7,94 654 630 6,12 597 566 1223 Bor 760 6B 610 586 588 5.54 524 12 175 asi 723 652 376 552 5.35 520 4g 13137 819 693 6.23 548 525 5.08 4.04 asa 14 1105 792 868 6.00 528 503 4.88 472 4 443, 18 1080 770 648 5.80 507 485 467 454 442 433 4.25 16 1058 751 630 5.64 491 469 452 428 427 448 410 17 1038 735 618 5.50 478 456 439° 428 414 405 397 18 © 1022 721 603 5.7 486 444 428 414 403 394 3.88 18 © 1007 709 592 5.27 456 434 418 404 303 384 376 2 994 699 582 S17 447 426 409 398 3.85 3:76 3.68 2 9.88 573 509 439 418 401 388 377 368 3,60 2 973 585 502 432 41 394 381 370 361 354 23 968 558 495 426 405 388 375 364 355 347 2% 855 666 552 480 420 399 383 369 359 3:50 342 25° 848 660 546 484 415 396 378 364 354 345 3.37 2 «841 654 51 470 440 389 373 380 349 340 3.33 2 834 649 5.38 474 408 385 3.88 2 © 828 644 532 470 4130 402 381 3,65 29° © 823 640 528 4668 426 398 377 361 30-918 835 S24 462 423 395 3m 358 32 908 628 517 486 447 399 368 352 34-901 622 511 450 41 384 363 347 3 «894 616 505 446 405 370 358 342 38-888 611 502 441 doz 375 354 339 40 883 607 498 437 399 3,71 351 3.35 42 Bye 434 395 368 348 332 “B74 431 392 365 345 320 4% © 870 428 390 362 342 326 4 88 425 387 360 340 324 50883 423 385 358 338 3.22 65 ass 418 380 353 333 3,7 6 89 414378 349 329 313 65 8.44 411 373 348 328 3,10 7 8.40 408 370 343 323 3.08 8 8.33 403 385 339 319 3.03 100 8.24, 396 359 333 313 297 125087 391 354 328 308 203 1508.12 388 351 325 305 280 200-806 54a 441 S84 347 321 301 286 400 07.97 537 434 378 341 315 295 280 1000 7,91 5.33 430 374 3.37 31 292 277 infrito 7.88 630428 372 335 309 290 2.74 228 Tabelas ‘TABELA A232 Distbuigdo F: valores eriicos para um test bilateral («= 0,01) (continuagao) giao do Numerator Denominador 7416202404080 —7S «100 ~—~200~—«S00_infno 1 ‘24987 25041 25146 25219 25795 25989 25900 25406 25468 2 1094 1995 199.5 1995 1995 190.5 1995 1995 1995 3 4262 42A7 42,31 42,21 4208 4202 4192 4187 41,83, 4 2003 1989 10,75 1967 1985 1950 1941 1998 19,92 5 1278 1266 1253 1245 12,35 12,90 1222 12,17 12,14 6 947 936 9.24 917 907 9.03 8,85 7 794 753 742 735 728 722 7.18 8 50 640 629 622 613 609 602 9 S73 562 552 545 537 532 526 10 8a7 507 497 490 482 477 471 " 478 465 485 449 440 4,96 4,29 2 443° 433° «423 «417 408 404 3.97 13 417 407 397 391 378 371 4 396 385 376 3.70 357 360 15 373 369 358 352 339 333 6 364 354 344 337 329 325 3.18 a 351 341 331 325 318 312 3.05 18 340 330 320 314 305 301 2.94 19 331 321 311 304 296 291 26 2 322 312 302 298 287 283 276 a 345 305 280 275 268 2 308 2.98 273 203 262 2B 302 2.92 2er 262 256 4 297 287 281 257 260 25 22 2a2 258 252 245 6 2s 277 252 247 240 2: a 283 2.73 2a 249 2.96 2: 28 278 2169 Das 299 292 228 225 29 2768 2.86 240 238 2.29 224 221 30 273 283 237 232 225 221 218 32 287 247 240 291 226 219 2a 34 252 242 235 228 221 2.14 2.06, 38 248 237 290 221 217 209 2.01 38 aa 293 227 217 212 205 97 40 240 230 223 214 209 201 1193 42 237 228 220 210 208 1:98 +190 “4 234 224 217 207 203 1195 187 46 232 221 214 205 200 1192 saa 48 219 212 202 197 1,90 fiat 50 216 210 200 1.95 1.87 178 85 242 205 495 1,90 1.82 4.73 60 2os 201 iat 188 178 163 65 205 1.98 1188183 1,74 165 70 202 198 185 1.80 171 1162 80 197 190 180 175 1/66 1156 100 4g1 184 474 168 1,59 149 125 186 178 4168 1.63 1153 a2 160 tiga 176 485 159° 1149 4.37 200 479 171 180 154 1144 431 400 173 185 184 147 136 121 1000 igo st 1160 143° 131 a3 infinito ier 15917140128 ot Tabelas 29 TABELA AG Distibuiggo F: valores critcos para um teste unilateral (a = 0,05) giao ‘ie Numorador Denominador 23 @ 8 6 7 8 3 ae 1 tei 199216225. 280-234-257 +—«29~—«24—«242 243 Daa 2 1851 19,00 19,16 1928 19,30 19.33 19,35 19.7 1998 1940 1940 10.41 3 1043 955 928 9,12 901 894 Bao O85 881 B79 Bre Bre 4 71 694 659 639 626 616 609 Od 600 586 Sor Sot 5 681 579 S41 519 505 495 498 482 477 474 470 5 589 5,14 453° 438 428 421 415 410 4084103 7 558 4,74 42 397 387 379 373 368 364 3.60 8 582 446 384 369 358 350 344 339 3.35 331 8 512 4.26 363 348 337 329 323 318 314 310 10 496 4.10 348 333 322 314 307 302 298 294 1484 3.98 398 320 309 301 295° 290 285 282 270 2 475 3.89 328 300 291 285 280 275 272 268 130467 3.81 318 292 283 277 271 267 263 2.60 14 460 374 ant 285 276 2.70 285 260 267 253 18 454 368 3.08 27a 271 Dee 250 254 251 248 16 4493.83 301 274 266 259 254 249 246 242 7 445. 1359 296 270 261 255 249 245 241 238 18 4at 385 293 26s 258 251 248 237 234 19438 382 280 263 254 243 242, 234 231 2 © 435 349 287 260 251 245 239 231 228 2 492 347 307 284 268 257 249 242 297 292 228 225 22 430 344 3.05 282 266 255 248 240 234 230 226 223 23 «428-342 303 280 264 253 244 297 232 227 224 220 2-426 340 301 278 262 251 242 236 290 225 222 218 2 424 «3.39 299 276 260 249 240 234 228 224 220 216 2% «= «423. «-33a7 298 274 259 a7 239 292 227 222 218 218 2 «424335286 273 2E7 240 Dar 231 225 220 217 213 2 420 3334 295 288 245 2398 220 224 219 215 212 2 418 333293 285 243 295 228 222 218 214 2:10 3 4t7 332 292 259 242 233 227 221 216 213 209 3 415 329 290 267 251 240 214 240 207 34 «413-328 288 2e5 249 238 212 208 205 3 41-326 287 263 248 236 ¥ 2it 207 203 38 440-324 285 262 248 235 226 219 214 209 205 202 40 408 323 284 234 225 218 212 208 208 2.00 4 © 407 322 283 232 224 217 211 208 203 1.99 44 4908 321 282 231 223 248 210 205 201 1198 4% 405 3.20 2.81 230 222 215 209 204 200 1.97 43 4043.19 2.00 229 221 214 208 203 199 1196 go 403 3,18 279 229 220 213 207 203 199 195 55 402 «3,18 277 254 298 227 218, 193, 60 © 400 «315 276 259 257 225 217 192 6 «399-314 275 251 236 224 215, 1130 7 398-313 274 250 235 223 214 189 8 = «396-311 272 249 203 221 213 1138 100 394 309 270 248 231 219 210 1.86 125 © 382 307 268 244 2.29 217 208 1183 160 390 305 268 243 227 216 207 1182 200-389 3.04 265 242 226 214 2.08 4180 400 3388-302 283-239 224 212 2.03 178 1000 385 300 261 238 222 211 202 176 Infiito 3.84 300 260 237 2.21 210 2.01 475 230 Tubelas TABELA Ad.1 Distbuigdo F: valores eiicos para um testo unitatoral (a = 0,06) (continuagéo) ‘ide Nomerador = Tabelas 234 TABELA A42 Distribuicgo F: valores erlicos para um teste unilateral (a = 0,01) ry ‘aide Namorador Denominador 72S 6789 @ 1 4052 4900 S764 5859 5928 5981 6022 e107 2 98,50 99,00 99,30 99,53 99,36 99,38 99,30 99,42 3 3412. 30,82 224 2781 2787 2749 27°34 27.05 4 2120 18,00 4662 1521 1498 1480 14/66 1437 8 1626 i327 10.97 1067 1046 1029 10,16 9.89 6 1375 10,02 87S 847 826 810 7.98 772 7 1225 955 748 7.49 699 684 472 ear 8 1128 B85 663 637 618 603 591 a7 8 1056 8,02 608 580 561 S47 535 eat 10 © 10.04 7156 564 839 520 508 494 ant 1 965 507 489 493 440 2 933 482 4.64 433 416 3 907 482 4.44 418 3.96 4 886 448 428 4.03 3.80 18 868 4320 4.44 3.89 37 16853 420° 4.03 378 355 7 840 40 3.93 368 348 18820 sot 34 360 337 19 394 377 352 330 2 387 3,70 348 323 2 381 364 340 331 328 3,7 2 378 359 335 328 3.18 332, 2 371 330 321 314 307 ca 328 3.17 309 3.03, 25 322 313 306 290 28 348 309 302 2/98 ar 315 308 299 2.93, 28 312 303 296 2/90 29 303 300 293 2.87 30 307 298 201 2184 2 302 2.80 Py 298 276 38 295 272 38 292 268 0 289 268 2 288 284 44 a4 262 46 282 260 48 Zao 258 50 278 256 55 275 253 60 272 65 268 70 267 80 264 100 259 125 255 480 253 200 2550 400 245 1000 243 infito zat 232 _Tabelas ‘TABELA AA.2 Distbuigdo F: valores crticas para um teste unilateral (a= 0,01) (continueco) giao 7 do Numerador Denominador W478 aD 7100 ~—~200 ~~ 800 _ininllo 1 6143 6170 6209 6234 4260 6286 6902 aaz4 6304 6360 6960 6366 2 9943 99.44 99.45 99,46 99,47 9848 99,48 9848 9949 99.49 99,50 99.50 3 26,82 26,83 26.69 26,60 26.35 2628 26,24 26,18 26.15 26.19 4 425 14,18 14.02 13,93, 1369 1361 1358 1352 13,49 1346 5 aT 968 988 847 924 917 9,13 904 9.02 8 780 752 740 7.31 709 7.02 8.98 690 6.88 7 836 628 616 S86 5,79 5,75 587 565 8 556 548 5,36 507 500 4.96 488 4.86 9 501 492 4381 452 445 4.41 43a 431 10 igo 452 at 412 408 401 393 391 n 429 381 374371 362 360 32405 357 350 347 338 336 133.86 338 331 327 349 317 4 370 322 315 31 303 3,00 18388 308 301 2.98 280 287 16 848 297 290 2.86 278 275 7 338 287 280 2.76 26s 2165 1 327 278 271 288 250 257 1 39 271 264 2.60 251 249 2 313 24 287 254 2s 2.42 2307 258 298 2.36 2 302 253 231 23 © 297 248 228 24 244 221 25 240 27 28 238 213 27 233 210 28 230 208 2 227 203 30 228 201 32 262 250 242 234 225 2,20 1,96 34 288 246 238 230 221 216 a1 38 254 243 295 226 218 212 1.87 38 251 240 292 223 214 209 201 a4 40 248 237 228 220 211 208 1.98 1.80 a2 248 236 226 218 209 203 1.95 178 44 207 201 1.98 175 48 20a 199 191 188 1.80 173 48 202 ter 189 184 78 70 50 2ot 1195 187 182 176 ‘is, 85 (242 197 191 183, a7 467 1.98 6238 i488 178 tes 1631160 6237 4g 4547 185 1160157 7 238 11g9 1183 1,74 1e2 157 154 Bo 231 ves 179 70, 158 15531149 too 227 x v0 474 152 1471.43 125-223-215 2.03 185176 1189 tar ua 137 180-220 212 2.00 193173 1,68 143138138, zoo 217 209 197 173 11691163 198133128 400-213 2.05 1192 175 184 1,58 192.6 125 1,19 4000-210 202 1190 172 161154 428419141 Infinto 2.08 2,00 1.88 1701891152 12515 1,00, siptoteca Universitaria UFSC ‘ Tabelas 233 ‘TABELA AS Distriouipdo binomial: probablidades para x sucessos em n observagies P n__x | 0050 0,100 0,200 0,300 0,400 0,500 0,600 0.700 0,800 0.800 0,950 20 | 0903 0810 0840 0,490 0,300 0,250 0,160 0.010 0.003 1 | 0085 o'7g0 0'320 0.420 O48 0'500 O80 0,180 9,098 2 | ofocs ofao alogo of000 01460 01250 0'360 0810 0.803 3 0 | 0957 0729 os12 0243 o216 0,125 0,056 0.001 1 | 0135 O43 o'saa ola O42 01375 0.288 9.027 9.007 2 | olor ofozr oss O89 01288 01375 Olas2 0243 9.135 3 O01 01008 O.027 ones 0125 0.216 0729 Oey 4 0 | 0815 0686 0410 0,240 0,130 0,083 9.026, 1 | Ort Oza o10 112 013468 0250 0,154 9,004 2 | dod dots O52 0265 O68 0375 O3te 0049 go14 3 oloo4 01028 01078 o'isd 0280 O3a8 02a O71 4 O.002 0,008 0,026 0,063 0,130 0656 O1615 eat) 0928 0,168 0,078 0.031 9.010 1 O410 01380 0.259 0.186 O07 2 01205 0309, 0.008 9.001 3 Oost o'ta2 D073 9.021 4 01008 01028 91328 0.204 5 0.002 01580 :774 6 0 4 2 0.001 3 0.002 4 Gost 5 0.354 0.232 8 oat 0735 7 9 | 9638 1 | 0267 247 2 | oa 318 3 | ooo a7 9,003 4 ‘oar 01023 5 (025 0,128 8 008 o372 7 0478 ao | ages 0.058 1 | 9270 01198, o,001 2 | ost 01298, ora 3 | aos 0254 oloar 4 01136 01198 5 O.oa7 oi2s4 6,005 8 01010 0.296, 7 o.001 01198, 8 1058, ois3 8 0 | 9630 9.987 0,134 0,040 1 | 0289 O87 oso o1156 2 | oss a172 O02 Ozer 0,004 3 | clos clos 0176 O27 Olo2t 9.903, 4 | 0001 0.007 joes of172 0074 9.017 9.001 5 C001 o017 o.ora 01472 0.088 9.007 9.001 8 0.003 o.G2t 0207 0,176 9.045 3.008 ? 1004 01267 9'302 9,172 9.063 8 01158 01302 9.987 0,299 8 Ola 0,134 01387 01630 to 0 | 0599 0249 0,107 1 | 0318 0367 0268 2 | 9075 o194 0/302 9,001 3 | o010 dos? o201 0,003 9.001 4 | 0001 sort ofoas lca 9.008, 5 O'001 01028 0:03 9.028 0.001 8 0,008, 01200 9.088 9.011 0.001 7 0.00% 0267 0.201 9.057 0,010 8 0233 0302 0,184 0,075 8 Oat 0268 O87 0.315 10 0028 0.107 0.343 0599 I 234 Tabelas TABELA AS Distribuigao binomial: probablidades para x eucesees em n observactes (continuaeS0) P n___x | 9.050 0,100 0,200 0,300 0,400 0.500 0,600 0,700 0.800 0,900 _ 0,980, it 0 | 0568 0314 0,086 0,020 0.004 1 | 01328 0384 0.236 0093 0.027 0.005 0.001 2 | 91087 0.213 0295 0200 o089 o1027 01008 9.001 3 | olora ori 201 o2s7 O17 O08: o'o23 O'os 4 | coor ois O11 olan 0255 Ofer Soro S17 0.002 5 0.002 0039 0192 0221 0226 O47 DOs7 O10 8 010 0057 O47 0228 0221 dase O38 7 0002 9.917 o.070 oe 0236 07220 ot a 004 01023 Oost 0287. 0221 8 oloot G.005 o'oa7 9/200 0.205, to O'0ot 0:00, 83 0.238 " o'aas 12 0 | 0540 0282 0069 o014 0,002 4 | 9381 O77 O06 0017 0,003 2 | 01099 0/230 28s olos¢ Glo16 3 | 017 oj0as O36 0142 O.05e 4 | oloc2 oz: O33 O213 Ota 3 ‘ofo04 01083 027 o:183, 8 0018 O77 0226, 7 6.003 O01 0.193, 2,004 8 2001 0,008 01042 O:124 0.021 8 0001 01012 1054 9.085 0.037 10 ofo02 o'0%6 230 0,090 n i003, 37 0341 12 1282 0540 16 0 | 0440 0.185 0028 o,003 1 | o37t 0338 0.113 01023 0,003 2 | ois 0275 o2t oo7s do1s o.o02 3 | 91036 0.142 0346 0,146 O'047 0.009 0.001 4 | 0008 0051 ¥ 5 | doot ota 8 003 ; 8 8 0 1 014 0,001 2B 01051 6,005 3 O42 01036 M4 O27 046 3 9329 O37 16 0185 01440 20 0 | 0358 0.122 1 | O37 o270 2 | o'e9 O65 3 | o(080 o't00 4 | 013 oles 5 | oloce dose 8 6,008 7 o.ooz 8 8 1 n 2B %3 0,002 4 9.003 18 0.032 0,002 18 0080 o.013, ae 01190 0.060, 8 0285 01189 18 0270 Os77 20 O22 01360 TTABELA AS Velores crtcos da dstibuigso qul-quadrad ( 7 ) Tabelas 235 a 0.20 00) 0.05) 0.02 0.01 (0,007 1 164 art 3.84 641 6.63) 1083 2 322 41 599 7.82 921 13,82 3 464 625 781 9.84 11334 1627 4 5,98 778 949 us? 1328 1847 5 723 824 07 1339 18,09 2051 6 858 10,64 12.58 15,03 1681 246 7 380 12,02 1407 1682 18,48 2432 8 11,03 13,38 1551 18,17 20.09 26.12 8 12.24 1468 16,92 19.68 2187 2788 10 1344 15,99 1631 21.18 23.24 2958 " 1463 17,28 1968 2262 2473 31,28 2 181 1855 2103 2405 26.22 3291 13 16,98 1981 2236 2547 2789 3453 4 18,15 21.06 23.68 2687 23,14 36,12 18 1931 2231 25.00, 28,28 30.58 37,70 16 2047 2354 26,30 2963 22.00 3928 7 218i 2a 2159 31,00 33.41 40,79 8 22.76 25.99 2887 3235, 34 4231 19 23,90 2720 30,14 33,69 35,19 432 2 25:08 2841 atat 35,02 37.57 48331 a 26,17 2962 2267 38,34 3493 46,80 2 27.30 3081 3392 37.68 40,29 4827 23 28.43 32,01 35.17 3897 4164 4973 24 23.85 3320, 36.42 40.27 4298, 51.18 25 30,68, 34.38, 3765 4157 4431 52\62 28 3179 95,56 3889 42.86 45,64 54.05 a 32,91 36,74 40,11 4at4 48,96 55,48 2 34,03, 3792 4134 45.42 4828 56,89 29 35,14 39,09, 4258 45.69 4959 58,20 30, 36.25, 4026, 4a77 47.96 50,9 59,70 236 Tablas ‘TABELA AT Valores erlcos da distibuigso g para a= 0,05 a ie 2 3 4 3 6 7 8 1 197 37084041 4640, 2 6.85 soles 174 13,03 3 7s02 8037 3,853, 4 3927 6287 6.707 3a? 5 3.695, 5673 6033 6.582 6 3.461 5305 5628 e122 0 3.344 5060 5,350 sais 8 3261 4808 8.167 5597 a 3.199 4788 5,024 532 10 381 4ess 412 5,205 " 3413 4574 4523 5.202 B 3.082 4508 4751 S18 3 3,055, 445341690 5.040 “4 3,033, 44074638 4990 18 314 4367 4595 4940 16 2,908 49934557 4.297 7 2984 43034524 4.358 8 2ert 4277 4495 aad 19 25960 4253 4460 4794 2 2950 42324445 4788 2 2919 41684873 4908 50 2308 4.102 4.502 40 2858 4038 4s2t 60 21829 397 aaat 120 2,800 33t7 4363, 2 2772 3.858 4288 (Gea 3s 4 6 6 17 1 50,59) 53,20 55,26 5722 2 14,39 15,08 15865 16,14 3 on7 40,15, 4083 1084 4 B27 8373, 3684 84 5 7.168 7486 71508 7.717 «71828 7.992 6 6549 7034 71437244 7,998 7 6,302 658 6759 6.852 6.939 8 6.054 6389 64836571 6.853 8 5867 6198 6276 8.359 8.437 10 6028 Bite © 6.194 6.269 " 5,605 5901 5984 6082 6,134 2 5511 5798 5878 6.023 13 5431 Syn 5789 Sge2 5931, 4 564 5,463 Ses7 S714 5,786 5.852 8 5308 5404 S574 8649 5,720 5,785 18 5255 5352 5520 5593 5662 5,727 7 8212 8307 Sai S44 5612 8675 18 Save 8267 528 S501 8588 5.690 19 5140 5,231 5301 5482 5528 8.589 20 5108 5199 5282 5.357 S27 84935553 24 5012 5089 S179 5251 5819 Saa1 5499 30 4917 S001 S077 S47 S211 «S271 8827 40 4824 4904 4977 5.044 5,100 51635216 ry 4792 41a08 aaa 4.942 S001 5058 8.107 420 4ea1 4714 4781 4942 498 4980 4/998 2 45524622 afea5 74s 4.7068 4.845 4.091 Tabelas 237 TABELA AT Valores citicos da distrbuiggo q para a= 0.05 (continuagao) k 0 2 2% % 2% ae 1 5988 6097 6522 6423 65,15 681 67.86 2 1677 7.13 Ws 1802 18.27 1872 i8e2 3 i128 tar 1137 1253 4 9233 adie 9,736 1034 5 5208 8.386 8.683 9,165 6 7,887 7,730 7979 3.452 7 7170 7/308 7533 T9872 8 6706995 r22 7625 8 6644 6.763 6370 71363 10 eas? 6582 erat 7188 " 6328 6436 6.628, 6.994 2 62086317 6.503, 6.358 3 et2 627 6.398, e744 4 6023 6.132 6303, Bear 6 59586059 ozs 6.564 16 5897 5.905 6,168, e4gt 7 Sea 5.940 e107 8 S798 5880 6.055, 18 S782 5546 6.003 20 Sita 8807 5.968 24 5594 5,683 5.838, 8.132 50 84755561 5703 5.900 40 S358 5439 5581 5.349 60 S241 8319 5453 5708, 120 51128 8200 5307 5.568 2 S012 5081 5201 5427 g |e 40 eo 1 68.92 7397 2 1928 2066 3 1287 1376 4 53 1124 5 9330, 9649 6 801 9.163 7 at0 8632 8 1758 8.248 3 7488 7958 10 7278 7730 1" 7546 7 71394 8 8.854 1287 4 8754 1153 8 8.669 7065 16 5.594 6.964 7 5520 912 18 Bart 6848 8 Bate ere 2 Bara 6740 4 8.228 9579 50 8.080 eai7 0 5.934 6.255 60 5750 8,780 6,093, 120 5807 S44 5,928 2 5463 6.498 5,764 Fort: Zar, 1989; ap. 64 (medica) 238 Tabelas “TABELA AB Valores crilicos da istrbuigdo U de Mann-Whitney, para testes unilaterais com a = 0,025 ebilaterais cam a= 008 1 2 o 0 09 4 4 4 4 4 2 2 2 2 3 2 3 3 4 4 5 &§ 6 6 7 7 8 4 4 58 6 7 8 8 0 MH H 2 1B 13 5 ie cee oleae vite toe see gif I 10g 108 19) 20 8 | 1 1% 4 1 7 19 2 2 BOF 7 2 4 1% 1% 2 2 2% 2% 2% 9 32 4 a | 1% 7 1 22 2 2% 2 3 3% 96 8B 9 7 2 3 2% 2 3 M&M % 3 4 46 48 3 | 0 2 2% 2 3% 3% 9% 42 4 48 82 85 " ose oe oO esa terrae Ae eT Sige co 982) Fee | one 0) oe lary ara to) ecole 0olee orga Sle Go ca 3 | 2 33 87 tS CGT TTB i | st 36, 40 «450 05ST HTB Spe oo 8 Oo Om we oe oC ws | a7 78 t]7SBL BW w | 39 45 st 87 63 OT 75 1B BD 10S Fi | preoeaa get see ogee orate rae soo 1 00) (0919 902 | t00) a2) jo | 40 52) 58 S72 BOSC HSC 55 Fonte: Siegel, 1086; p. 276. Tabelas 239 TABELA A® Valores coos da dstibigdo T de Wecovon nm | aBlatek 050 020 040 008 002 001 0008 oon aUniaters 0.25 0100.05 0,025 O.01_—0,005 0025 0.0005. z 2 ° 3 4 2 ° 6 8 3 2 ° 7 8 5 5 2 ° 3 2 8 5 5 4 ° 8 16 10 5 5 3 4 ° 10 20 ia to 8 5 3 4 ca m4 7 13 +0 7 5 3 ° 2 2 a sid 3 8 7 5 4 8 3B a v 2 8 7 2 i a i 2 2 5 e 3 4 18 a7 8 50 2 8 8 2 5 8 54 ag 35 2 (8 19 8 8 v a 6 a as rae 18 n iB s 88 a © 2 2 cr to 7 @ 5 a8 sO 7 ct 2 a a8 ®0 2 4 2 OF 2 ost ger 7 5 49a 7s 2 im a 75 a3 5 50 2 M4 84 83 73 & 4% EY ba 2 108 ot a 60 i a 25 13% 03 t00 8 8 8 28 48 me Ho 4 7% oar 51 a jo i343 tor aw 33 74 a7 28 ie eG eae aa 39 1 ts} zeta 50 cf a 8 31 2120 ist e370, tna ws 2 zeke] Ot 8 re eC re a 2 i 208k tas Ht 35 Pee ee Poe et te er ar) | 36 29 250 eT 208855720 a 30s 265] 2k 888880 38 ee re ee re 3 Bo orn an at 40 SSB ee see ge 2s ot eet ened re 4“ 3738025, a Sos 319 2s 285 Ba? 28088 % a6 368310281 281s 207 a 450 38s ar 28328253220 a5 eae ee rN oe) et et ear c 8 eri ar eed a sed ae ee aren cere te a7 439 ai M7, a4 BS 280 4 sot ez ae 508258931827 Pi S43 asa igs 7838828 50 Eee ce re i 51 90 625405 ds3 at 3008719. 5 eis 547 507,385 53 633 foo B20 aaa aa 5a ee 55 Cr | 58 M4 638505 57th ae 870 5 Ho Sea gis 88 Bk ar 80, 58 a ne ae) 53 rie ee Gal a eee art 60 bez 7800s 00 BBS 240 Tabelas TTABELA AQ Volos citcos da distrbuigdo T de Wilcoxon (continuago) 2 | aBiistere: 050 020 010 005 002 001 0,008 ooo Unite: 025 010 O05 0025.01 _—0.008 0.0025 0,005 ft a0 765—~=~*STSSC*«ia TSC S*«iOSCSC«CSCSC*« 62 a ce rr rT) 63 s08 819 7e7 72k wD, 64 s8 077837? 65 98 8757S TST 68 998 903, aay 708742705780 87 102903275788 729 68 105i 96280378] TSA TB a 69 109302317777 8G 70 1126 1022 «980 807,857 BD n ‘159 105398083 wat mH n ‘g2 1084 = 1020S 64 = OTST a 1260111610504 gw m4 12611148 fost 023572788 5 129001181112 1053688 HOB 76 133112141144 toa = 1015 68256 7 1987124711785 1088S a7 asa 8 14031282 1209147, 10751028 B18 73 1440 131612421179 11051058 «0106 80 w7e 1351127621136. 1086 109A, at 1516 13871310 t2ag 1188116059 2 155414231345 a7? 1200S 147 «=o» a0 8 1598 1489138031 125211781129 1028 a4 1632 4961415151285 1210116010857 85 1e72 15831451. 13801288 = 12821811086 88 17121871 87 aS 1352 17S taes att a7 W753 16091624481 136613081255 1148, 88 1794 16481561487 ©1400, 1342288 HHT a9 jase 16881589528 1435 | 13763211208 90 sere t7aT=— 163815501471 HT 1855 1240 st 1921767 167815071507 1445989274 92 1964 18081715835. 154314801423 1304 3 2008 = 1843755 T6748 1580 51614581397 94 ost i891 17851712 tT 882— 14931370 9 2087 1833S 183817521655 «15891529104 96 zz 1976 «18771701693 162815851438 or er 201918 183217311884 = tot tare 88 2233 20s 186018721770 «170216381807 89 2200 © 210620319813 1810740 16781543 100 Zoot 261 ams 10551850 7797141878 Font: Zar, 1058, ep. 109-102 (roccad | | TABELA At0 Valores ertoos da distibuigdo He Kruskal Wats Tabelas 241 040 4571 4285, 41500, 4srt 4350 4922 4500 0,05 4945 8410 5625 5724 8765, Bagot 5819 5.305, 5679 ener 5833 0.02 7.332 1355 (6667) 6500 01 0.005 7.200 0,002) 8.048 att Saat 8,868, ‘9620 0008 8,909 9269 9.348 8827 e170, Beet 9.109 to2r1 9,692 9752 $6480 t01342 101524 10,888 t1310 41,705 242 Tabelas -TABELA A10 Valores cricos da dstibulcao H de Kruskl-Walis (continagdo) my Pe Py fe 0.0 0,08 0,02 or 0005 0,002 0.004 322 2 be eams8S787.189~— 7.889 3.314 5338 (8,333, 3321 5689 6244 = g.689 7.200 7.400 33.22 B75 GST 7.182 7.836 7.873. 8.455 3334 Bess 660071097400 8.08. 33.3 2 Bers G27 7638805. B.379 9.030 3303 3 Boze 700078728538 B88 9513 4444 = 4245 5,280 ee alezeat 5.533 esr 7,000 4.2.22 yost get 7,884 8.201 4344 5067 ern 7087 ae sepel 5591 7018 745577738, 182 4422 5750 7s Teri = 82738689 8.908 ry ed soe G85 7485 775B 8.212 BN? OAR miasushe2 5a72 8798 7.763 333 By1e 9,167 9455 43°33 sors = 884 7.095 BSD «9.253870 10,016 44414 592-5945 = 7001 7,909 7,009 4424 3568 6386 7.364 7885 B.34t 8,909 4422 eos «6731 «7750 «8348 8.802 9.462 4434 Beez 6635718608231 8.589 9.327 aa se) sot 6.874 7.051 e219. 165 9,045 4433 6019 7.038 81818878 (8.405. 10.467 4444 Bee 67257879 «.588 (9,000 9,758 4442 Soi 69578157 B71 B48 i 16,428 444 3 eo © 74428350 8075 9.742 10.542 10,929 4444 60s «7235S 8287 G.B7t_ 10.809 1,.098 24.19 = 221114) 5786 222 11) 6250 6750 22 2 2 1) ee 867133738) 7,838 322 2 2) 69e2 7418 «Bors, 8.201.727) BTR Bete = 321 71) 61e 6589 3 2 214) ss 6900 7400 7.600 BAH OO te ety ee 322 2 2) @oss 7osz 8903 Beez 8885 9.273 9.964 see teri este a7 iil aaea7 sr 33.2 1 4] 6e00 7200 70928073 8.848 332 2 4) 67e8 7501 8.258 «8675 Gat G67, 8.808 3-32 2 2) 728 790 8867 11574 8.789 10.026 3303 1 1| 678 6576 8242 B24 88AB (BASS) 8.ASS 333.24] 690 asso 9051 at ren 9.874 33.3 2 2] 712 sort 9059890 «10,830 +10,637 sess sets et er 0rr Bera ast 84510286 10.549 393.3 3 2) 720 9267 = ga7a 108831083817 isles ale sets aioe| tT 225 e467 10.200 —10.733—10.267___—11,867, Fonte Zax, 1000; ap, 108-106, Tabelas 243 TABELA Att Valores cricos da cistibuipdo @ para testes de comparagées maliplas ndo-paraméticas 020010 00502 001 ~~—«0.008 ~~—~0,002 ~~-001 1282 4.645 —*1,960=—«aar 2876 2.807 30814201 1834-2128 2.394 2713 2.998 34d 3,403,888 2128 2394 © 2a39 2883445342 3688) 3.765, 2827-2576 207 3.081 az01 ast «3719 «Saat 3588 3620 3.988, 3902 4067 3748 3972 4.134 3a10 4031 4.191 3965 4083 4.241 24752713 2938 3.209 2593 2823 30u8 «3.304 2090 2814 3128 3.384 2773 2992 3197 3489 2948 3059 32613612 2908 319 3317 3.565, 2965 3172 3968 3.613, 3018 3.220 3414 3.856 3062 3.264 ©3456 3,605, 3108 330434043731 3144 3342 3.520 3,785 381-3376 geez 3.798, 3215 3409303 3.825, 3248 3493622 3.852 3278 3487 36403878 3904 3494 3.678 3.02 3331 3519 3.609 3.925 3355 3583 3722 3.947 3980 35063744 3.968, 34033683765 3.988, 3916 4123 4.286 3957 41714328 3997 4208 4.363, 4034 4244 4.307 4067 4278 4.428, 4098 4.908 4456 4527 4.383 4483 4184 43359 4.508, 41734303 4.502 4203 4408 4.854 4728 44284875, 4207 4.448 4505 4268 4458 4614 4287 4488 4,692 43054504 4,649 Fon: Za, 1999; ap. 107 244 Tabelas ‘TABELA A12 Valor crtcos para 0 cosficente de carrelacso de Spearman (r,) para postos f [aBlaterat 050 020 005 002 001 0005 0,002 0,008 JaUnilsersl: 0.25 0.10 0025 0.01 0.005 010025 0,001 _0,0005 a (0600 1,000 8 0500 0,800 4,000 1,000 6 oar! 0657 0888 0.843 1,000 1,000 7 ogat osrt O7as 088301923 0.864 1,000 8 0310 0735 0833 08s1 0.905 0.952 8 0267 0700 078308830867 0.817 10 0248 oles 0745 0,794 0830 0.879 " 0.236 0618 0,709 0755 9.800 0,845 2 o2i7 0587 0678 oar ores O88 8 0,209 80 0648 «0.703 O77 O79 4 0.200 0538 0628-0878 0.723 O77 18 O89 O21 604 ©0654 0,700 0,780 16 o,te2 0503 0582 0635 0679 0,729 7 076 0485 01686 OB1S 0662 0.713, 8 0170 0472 0,550 0600 0643 0.005, 10 0.165 ose 0628.67? 2 o.t6t 0570 0812 o682 2 0,158 0558 0599 0,648 2 0.152 og4e 0586 eae 2 0148, 0532 0573 0622 4 onas Og2t 0562 0.610 25 o142 Ost 0581 0.598 6 0.138 0501 os4t a 0.136, ost 0531 2B 0.133, 04530522 0.567 2 30 0475 0513 0.858 80 0.128 0487 0,508 0,549, 3 0.126 0459 0496 0.541 32 o.t24 0452 oa89 0.533 33 oat 0448 0482 0.525, 34 0,120 0439 0475 OI7 35 018 0433 0.488 0.510 98 0.116 0427 0462 0,504 a7 Oat 04580497 38 0415 0450 0491 39 oai0 0.485, 40 0.405 oro 4a 0400 04930473, 2 0395 0428 0.468 “3 ogst 0423 0.463, “a 0386 o419 0.458 45 ogez 0414 0.483, 48 0378 o410 0448 aT O37 0405 0.443 43 0370 0401 0.439 49 0368 0.397 0.434 50 0363 0,383 0.430, st 0359 0300 0.428, 52 0356 0.388 0,422 53 0352 0382 0418 54 0349 0379 O44 55 0346 0375 ott Tabelas 245 TABELA A12 Valores crlcos para o cocfcionte de corrlagSe de Spearman (,) pare postos (continues) n [asiitera: 050 020 010 005 O02 JaUniaterat 025 010 0,050,025 0,01 56 0082 0174 O2a2 0284 ogit 7 0091 0,172 0220 9261 0/308 58 O09 O71 o218 258 0.306 59 0.089 069 0216 257 0.303 60 0.089 0,168 O24 0285 0.300 6 0.088 0,166 0213 0252 0208 82 0087 0,165 O21 0250 0.208, 53 0086 ©0163 0.209 0248 © 0.208 & 0086 0,162 0.207 O25 0.281 65 0.085 O61 0,206 0,289 6 0.084 0,160 o2e7 er O08 0,158 084 68 00830187 oze2 cy oo 01156 0.280 0 0820185 o27e 7 0081 0.184 o276 n 0081 0.153 ora i 0.080 0.182 0272 7% 0.080 0,151 oar 6 0.079 0,180 o,191 0260 78 0078 0,149 0,190 0207 7 0078 0148 0.189 0.265 B 077 oj147 ote. 0264 a O07? 1145 0186 0,282 80 0076 0;145 01185 0.260 81 076 o,t84 2 ou7s 0.183 3 ovt82 a4 oat 85 0,180 88 0,179 er or? 88 0176 20 o.r7s 90 oa ot oars 2 O73 % ore 94 ort 95 0.170 96 0,169 7 01168 88 0.167 s0 0,168 100 01165; Fonts: 2, 1000; ep. 116-197 (odio) 246 Tabelas TABELA 13 Nimeros aleatérios 14095 07962-28733 66337 20020 46848-24360 «7813. «17531 98160, 84156 22328-08704 © 08439-4789 19608 74597 4289996235 1088 743984938. «67799 «7849303976 «72783 «31131 GoSZ 29880 BB2I2 0562 0649958274 8952877248 © 82823-29149 02415 aed © 34972 92554 02182 «68212-23811 «74309 «O1856 «50828-05868 «GOTT -36120 87154 33490 44547 19479-28029 «9873502523. «07352 «26115 —_OsTaM 30592 95545 09878 «3929105498 «2061813925 88848 OSTST_ 10298 63113 59195 90890 S294 6027 BEES 7615D 0102-23247 38135. 2456 1526100582 «37612 «T1971 9284444112 «4816115428 26704 920277388 «51468 91049 «19804 02188 «13918 ©2280 g4B5D S245 e891 23578 84958 96620 99600 94748 42738 «$7578 790637765 84885 80345. 9601S 01251-09848 ©8560 TITAT©~— TEST OTS 3642 30697 6938998345 73043-29507 1852667738 88ST 4974802160, ‘39871 02677 «13729 60302 «49365-36310 «29228 ©«52028 93731 $8465, 33006 74668 «41831 49768-95000 21405 «32144 NEAT © Gadd © 36257 o7154 82834 40789 1042281214 © 26025 @5495 | 48348 27904 © 76206, 31432 17859 22068 ©4194 © Gees © 348BBGST6S | «25467 35774 1056, 56960 26038 «3663291651 29180 98155 0180S S1464 © 49138 05710 02385 56388 09067 75595 25493 ST169 2288821475 «3111053048, 82103 63195 65527 6624396807 GaTES © 95289 628306634387 50825 82955 24147 75012 20103 «60287 04051-11854 © IASB 02020 03628 5542772771 «11270 «13301 «42087 «25848 «80ST 39840 34384 488e1 9526289450 1008792371 99885 94941 «46284 «77397 40100 sot! 4440192573 B4BZ1 49314 © 3434201200 «9116337248 35041 52043 24172 16959-76008 «4121 ««get99 «55271 38518 OTIS «97528 45342 3410348817 5363503630 g0499 © 17001 779T1 8790091034, Tae 27536 54074 82623 6492232241 80784 1459017868 —72187 77329 75480 19088-91100 «21175 a7a6o 8479 © 87906 3006814348 35196 84012 03780-47762 «ads «BUBI2 71238 ©« 5407043360 «61385 B4371 38352 «85742 0161041863 50977 5851379876 «7152 0249 22980 08123 9809335609 4540857014 98884 29851 6597903903 58486 17927 9t107 8300280223 «04731 «880395720 91892 1246 61376 95034 53855 2967013902 67790 92887 «6972598265 90459 25758 3557507730 «38317 «40512 -« 9504186943 68526-24735 38609 04044 43464 90762 «94781 © 68427 «50021 8290833059 41037 Ba4T7 60047 5088164384 42320 aa01s ©1491 23858 55507 47947 10069 83631 8623540884 4840096307 3728506290 04315. 0577318621 06644 9290769791 $4410 6316131227 «T0873«B7O1T «8948309370 497a1 25181 2980548135 94955 7642 «45637 «2820077295 40800 21205 6144244858 73413 T9504 58741 g02TB_ 78053 -247ED 77854 Referéncias bibliogrdficas ACHUTTI, A.C. et al. Hipertensto arterial do Rio Grande do Sul. Boi. Satie, Secretaria da Saide Meio Ambiente do Rio Grande do Sul, v.12, n.1, . 6.54, 1985, AZEVEDO, M.I Hiperfiltracao glomerular em pacien- tes dlabéticas insulino-dependentes: aspectos evo Iutives, patogenéticos e cardiovaseulares. Porto ‘Alegre: Curso de Pés-Graduagao em Medicina/Cl nica Médica, Universidade Federal do Rio Grande do Sul, 1992. (Tese de doutorado,) BARBOSA, A.C. et al. Hair mercury speciation as function of gender, age, and body mass index in inhabitants ofthe Negro River Basin, Amazon, Bra- zi. Arch. Environ. Contam, Toxicol, 40, p43-444, 2001. BARROS, FC. et al. Satide perinatal em Pelotas, RS, Brasil. Patores socias bioligicos. $0 Paulo: Rev. Saide Pil, v.18, nA, p301-312, 1984, BAU, CHD. et al. Heterogeneity in early onset of ‘coholism suggests a third group of alcoholics. Al ‘cohol, v.23, p.9-13, 2001 BODANESE-ZANETTINI, M.H. etal. Aneuploidy and ‘chromosome mosaics in hexaploid wheat (Tritium ‘aestivum (L.) Thell) cultivars. Cereal Res. Com- ‘mun. 21, m4, p.269-275, 1993. BONORINO, C.B.C:; VALENTE, VIS; CALLEGA- RI-JACQUES, SM. Urbanization and chromoso- ‘mal polymorphism of Drosophila nebulasa. Rev. Bras. Genét. v.16, o, p. 59-10, 1993, BORGES-OSORIO, MLR; ROBINSON, WM. Gend- ‘ica humana, Porto Alegre: Artmed Editora, 2001. BORGES-OSORIO, MRL. Fatores genéticosedo.am- biente nos distirbios de linguagem. Porto Alegre: Curso de Pés-Graduact0 em Genética, Universi- dade Federal do Rio Grande do Sul, 1985, (Tese de doutorada,) BRITTO, CM.C;, MELLO, M.L.S. Morphological di- ‘morphism in the ¥ chromosome of “pé-duro” cat te in the Brazilian state of Piaui. Genet. Mol. Bil, v2, p.369-373, 1999, CALLEGARI-JACQUES, 8.M.; SALZANO, FM. ‘Genetic variation within two linguistie Amerin an groups: relationship to geography and popu tion size. AmLPhys Anthropol, v.79, p.313-320, 1989. CARVALHO, PR.A. A lnfecceao como fator de risco ‘para colestase associada & nurigdo parenteral en trlangas com Idade superior a um més. Porto Ale- ‘re: Curso de Pés-Graduaclo em Medicina: Pedi tria, Universidade Federal do Rio Grande do Sul, 1993. (Tese de doutorado.) CCRUSIUS, C.A. 4 razdo como faculdade caleuladora =A “Aposta” de Pascal. Porto Alegre: Ed, Univer- sidade/UFRGS, 200 DANIEL, W.W. Applied nonparametric statistics. Bos- ton: Houghion-Miffin, 1978. DICIONARIO de especialidades farmacéutica. 26, ed, Rio de Janeiro: Editora de Publicacdes Cientificas, 1997-1998, DOLL, Rj BRADFORD-HILL, A. A study ofthe ati- ology of carcinoma of the lung, Brit. Med. J. ¥. 4797, p.1271-1286, 1952. DORNELLES, CLL. Yariabilidade genética protéica em populardes caucaséides do sul do pais. Porto Ale- ‘re: Curso de Pés-Graduacto em Genética e Biolo- ‘ia Molecular, Universidade Federal da Rio Gran- de do Sul, 1998. (Dissertacao de mestrado.) DOULOT, FN; LOPEZ CAMELO, JS; von GURA- DZE, H.N. Analysis of sister chromatid exchanges (SCE) in human populations studies. Rev. Bras. Genét, v.15, n.l, p. 169-182, 1992. DUCATI, A. PITONI, VILL. Morfoconquiliometria e Drymaeus (D.) papyraceus papyrifactus Plsbry, 1898 (Gastropoda, Pulmonata). In: Livro de resi ‘mos, VII Salto de Iniciagzo Cientfica da UFRGS- 1995, Porto Alegre: UFRGS, 1995. p. 118. DUTRA, 1. et al. Heterozygote detection in two hyperphenylalaninemia types: classic phenylke- tonuria and dihydrobiopterin biosynthesis defi- ciency. Rev. Bras. Genét, v.9, nl, p. 123-131, 1986, EDWARDS, A.L. Experimental design in psychologi- cal research, 5.ed. New York: Harper & Row, 1985. EVERITT, BSS. The analysis of contingency tables. 2.64, London: Chapman & Hall, 1992. FERREIRA, A.G;, CALLEGARI-JACQUES, S.M. feito da estocagem sobre a germinagao de Mimo- ‘sa bimucronata (DC )OK. e Leueaena leucocephala (Lam, de Wit Cién. Cult, v32, n8, p. 1069-1072, 1980. 28 Referencias Bibliogréficas FISHER, R.A. The design of experiments (Reimpres- ‘sto da 8° edigo de 1966). New York: Ha ner,1971 FLEISS, LL. Statistical methods for rates and propor tions. 2.04, New York: Wiley, 1981 FLORES, RZ. etl. Quiromanci el es su poder de ‘prediccidn? £7 ojo eseéprico (Buenos Aites), v9- 10, p.36-39, 1994 FOGLIA.VG. etal. La diabetes por pancreatectomia ‘en a tortuga normal ehipofisopriva, Rew. Soc. rg. Biol, v.31, 1-3-4, p. 87-95, 1955, FREITAS, TRO; LESSA, EP. Cytogenetis and mor- phology of Ctenomys rorquatus (Rodentia: Octo- dontida). J. Mann. v.65, 04, p.637-642, 1984, FUCHS, RD. etal. Effect of sodium intake on blood pressure, serum levels andenal excretion of sodium and potassium ia normotensive with and without familial predisposition to hypertension, Braz. J. Med. Res, v20, p.2534, 1987. GIBBONS, J.D. Nonparametric statistical inference ‘New York: McGraw-Hill, 197 GIRARDE-DEIRO, AM. Influéneia de manejo, pro- JFindlidade do solo, inclinacao do terreno e metals ‘pesados sobre a estruturae a dindmica da vegeta- a0 herbicea da serra do Sudeste, RS. Porto Ale fre: Curso de Pés-Graduacfo em Botésics, Uni- Yersidade Federal do Rio Grande do Sul, 1999 (ese de doutorado.) GROSS, JL. etal. Rsk factors for development of pro- teinuria by type Tl (non-insulin dependent) diabetic patiens. J) Med Biol. Res, v26, p.1269-1278, 1993 HEIDRICH, EM. letereia hemoliiea neonatal: est dos genéticos ¢ epidemiolégicos. Porto Alegre: ‘Curso de Pés-Graduagao em Genética e Biol. Mo- lecular, Universidade Federal do Rio Grande do Su, 1992. (Disseriagso de mestrado.) HOEL, PG Estatistica elementar. Rio de Janeio: Fun do de Cultura, 1963 KASNER, E} NEWMAN, J. Mateméiiea ¢ imagina- ‘0 2, Rio de Janeiro: Zahar, 1976. KIRKWOOD, BR. Essentials of medical statistic. ‘Oxford: Blackwell, 1988, KRAHE, C, Caracteristicas associadas & densidade ‘mineral éssea em um grupo de mulheres pré-me- nopéusicas. Porto Alegre: Curso de Pés-Gradua- ‘glo em Medicina/Clinica Médica, Universidade Federal do Rio Grande do Sul, 1995. Dissertg0 ‘de, mestrado.) LISBOA, HK et al, Clinical examination is not an accurate method of defining the presence of goitre in schoolchildren, Clin. Endocrinol, v.45, p. 471~ 475,196, MACHADO, V; ARAUIO, A.M. The colour polymor- phism in Chauliognathus flavipes (Coleoptera, Canthardse).1, Geographic and temporal variation Evoluciin Bioidgica v8.9, p-127-139, 1994-1995, MAISTROV, LEE. Probability theory ~ 4 historical ‘sketch, New York: Academie Press, 1974. (Tad do original em russo, de 1967). MALUF, $.W; ERDTMANN, B. Evaluation of occu- pational genotoxic risk in a Brazilian hospital. Ge- het. Molec. Biol, v.23, p.485-488, 2000. MANTELATTO, FLM.; FRANSOZO, A. Crescimen- to relativo e dimorfismo sexual em Hepatus pudi- ‘bundus (Herbst, 1785) (Decapoda, Brachyuta) no litoral norte paulista. Sa0 Paulo: Papéis Avulsos Zool, 39, p. 33-48, 1994 MATTOS, T.C. Estudio morfolégico das regides orga- nizadores do nuciéolo em neoplasias da cérvice ‘lerina, Porto Alegre: Curso de Pés-Graduagio em Genética e Biologia Molecular, UFRGS, 1994. (Dis- sertagdo de mestrado.) MEDEIROS, LF. etal. Electroferograma do soro san- ‘elineo de eqiinos normais da raga puro sangue ‘Mangalarga de 1 2 anos de idade. Rev Bras. Bol, 37, p£175-178, 1977. MELO, 8.4. Dindmica da capacidade de neutraliza- {900 de dcides (CNA) nas lagoas marginals da pla- nicie de imundagdo do Rio Mogi-Guacu (SP). S30 Carlos: Curso de Pés-Graduacao em Ecologia e Recursos Naturais, Universidade Federal de S30 Carlos, 1993. (Dissertacdo de mestrado.) MENNA-BARRETO, ¥; ARAUIO, A.M. Evidence for host plant preference in Heliconius erato phyllis from southern Brazil (Nymphalidse). J. Res. Lepi- doptera, ¥24, 41-46, 1985 MICHELON, E; MORIGUCHI, ELH. Caracteristicas da Aistibuigdo dos lipideos plasmaticos e dos ftores de isco coronariano em individuos com 80 anos ou mais. Porto Alegre: R-Med. PUCRS, v6 p-13-23, 1996. MORISSO, E-D.P; KRAUSE, L. As conseqiéncias do ‘manejo sobre os nishos de Dermochelys coriacea (Linnaeus, 1766), junto 20 projeto Tamar-Tbama, Espirito Santo, Brasil. Cuad. Herpetol., v.15, 2.2, 91-106, 2001, MOURAO, LAC, Sensibilidade gustativa & fenil- tio-carbamida e outras caracteristicas em iuber- culosas e ndo-tuberculosos de Fortaleza, Cearé. Porto Alegre: Curso de Pés-Graduacdo em Genéti- cca, Universidade Federal do Rio Grande do Sul, 1975. (Dissertagdo de mestrado.) MUNOZ, N. etal. Hot maré drinking and precancer- ‘ous lesions of the oesophagus: an endoscopic sur- vey in Southern Brazil Jat. . Cancer, v39, p.78- 708, 1987, NAOUM, P.C.; MOURAO, C.A.; RUIZ, M.A. Altera- ‘gbes hematolégicas induzidas por poluigdo indus- tial em moradores ¢ industrétios de Cubat3o, SP (Brasil), So Paulo: Rev. Satide Pabl.v8, 0.4, 271-277, 1984, NASCIMENTO, LC. do, Reguladores ontogenéticos ‘para velocidade de desenvolvimento e sua relagao ‘com a longevidade en Drosophila melanogaster. Porto Alegre: Universidade Federal do Rio Grande do Sul, 1992. Dissertagdo de bacharelado em Cign- cing Biologicas: Genética.) OLIVEIRA, MLB. Padbdo de pressdo arterial sistélica em reeén-nascidos normals através do método dop- ‘ler. Porto Alegre: Curso de Pés-Graduagao em Me- dicina/Clinica Médica, Universidade Federal do Rio Grande do Sul, 1994 (Disseragao de mestrado,) OSBORN, JF. Statistical exercises in medical research (Oxford: Blackwell, 1979 PALATNIK, M; SILVA, VIF. ABH-Lewis blood groups in captive Rhesus monkeys. Rev. Brat. Gendt, v9, 4, .679-84, 1986, PALATNIK, M. etal. Genetic and taxonomical aspects of Biomphalaria glabrata ABO agglutinins. Rev. Bras. Genét,, v3, nA, p375-386, 1980. PEARSON, ES. The probability integral of the range in samples of rn observations. from a normal popt- lation, I. Foreword and tables. Biometrika, v.32, 301-310, 1942, Apud ZAR, I. Biostaistical analy sis. 4.ed. Upper Saddle River - NJ: Prentice-Hall, 1999. PEDROLLO, F. Detecedo de hemoglobina Bart efie- Agilencia de falassemia alfa em wma populagao de nneonatos de Porto Alegre. Porto Alegre: Curso de 6s-Graduaglo em Genetica e Biologia Molecular, Universidade Federal do Rio Grande do Sul, 1988. (Dissertgo de mestrado,) PEDROLLO, E.; HUTZ, MH; SALZANO, FM. AL pha thalassemia frequency in newborn children from Porto Alegre, Brazil. Rev. Bras. Genet, v.13, 13, p.S73-581, 1990, PESSOA, LM; REIS, SF. Natural selection, morpho- logic divergence and phenotypic evolution ia Pro- cechimys dimidiauus (Rodeatia: Echimyidse). Rev. Bras. Genét, v.14, p. 705, 1991. PINHEIRO, CEA. Peso ao nascer na espécte huma- nna: 1m enfoque multifatorial. Porto Alegre: Curso de Pés-Graduaeo em Genética, Universidade Fe- eral do Rio Grande do Sul, 1989. (Dissertagdo de mestrado,) PINTO, LLB. etal. The frequency of genetic diseases ina high risk ward ina pediatric hospital. Rev Bras. Genét., v.19, 0.1, p. 145-149, 1996, RAHE, A.J. Table of eiticl values for the Pratt matched pale signed rank statistic. Amer: Stats. Assoc, ¥.69, 368-373, 1974, RICE, WR. Analyzing tables of statistical tests. Bvo- lution, «43, 0.1, p.223-225, 1989. ROBINSON, WM. Associacao entre trombose veno- +4, sistema sangiiineo ABO e pardmetros hemosté- licos. Porto Alegre: Curso de Pés-Graduagao em Genética, Universidade Federal do Rio Grande do Sul, 1974. (Dissertagao de mestrado,) ROCHA, G.A. PENA, S.D.J. A quantitative study of human amniotic fluid fibronectin in the second ti- mester of pregnancy. Rew. Bras. Gendt, v.10, nal, p.119-125, 1987, i, ROMANOWSKY, H.P; GUS, R; ARAUJO, A.M. ‘Shudies onthe genetics and ecology of Welicaninserato (Lepid.; Nymph). II. Population size, preadult ‘mortality, adult resources and polymorphism in natural populations. Rev. Bras. Genet, v. 45, n4, 563-569, 1985, Referéncias Bibliogrficas 249 ‘SALZANO, FM; SUNE, M.V; FERLAUTO, M. New studies onthe relationships between blood groups and leprosy. Acta Genet Stat. Med, v.17: 530-544, 1967, SALZANO,FM. et al. The Caingang revisited: blood sgeneties and anthropometry. Am. J. Phys. Anthro- pol, v.53, p513-524, 1980. SCHAFER, A. etal. Estudo comparativo da variagto didria de oxigenio em lagoas do Banhado do Taim, RS. NIDECO, Porto Alegre: Instituto de Biociéa- cias/Universidade Federal do Rio Grande do Sul, v4, p.5-38, 1980. (Série Taim) SCHERRER, IF etal. Susceptibility of Biomphalaria Tenagophila hybrid descendants to two stains of Schisiosoma mansont. Rew Bras. Genét, v.13, 23, .459-476, 1990. SIEGEL, 8. Nonparametric statistics for the behavioral sciences. New York: McGraw-Hill, 1956, SILVA, B.TF. eta. Protein electrophoretic variability ‘in Saimiri and the question of its species status. Aim. J Primat, v29, p.183-193, 1993. SNEDECOR, GW. COCHRAN, W.G. Métodos esta- disticos. México: Compania Editorial Continental, 1971, SOARES, PRB. O alelo DOB *03, 0 DNA do papilo- ‘mavirus humano e a precocidade da iniciagao se- xual en relagao & menarca, como marcadores naneo- plasia intz-epitelial cerca, Porto egre: Curso de Pés-Graduagdo em Medicina, Pontificia Universida- 4 Catdlica do Rio Grande do Sul, 1998, (Tese de dou- torado.) SOKALRR; ROHLF, FJ, Blometry: 2 ed. San Fran- cisco: Freeman, 1981 SPRENT, P; SMEETON, N.C. Applied nonparametric statistical methods, New York: Chapman e Hall / CRC, 2001. STEIN, ACR. Extudo comparativo e evolutivo de pa- Cientes diabéticas com diferentes graus de protet iniria, Porto Alegre: Curso de Pé©-Graduagio em ‘Medicina: Clinica Médica, UFRGS, 1984, (Disser- taglo de mestrado.) SUISSA, Sj SHUSTER, J.J. The 2x2 matched-pairs ‘wal: exact unconditional design and analysis. Bio- metrics, 47, 0.2, p.361-372, 1991, ‘TESCHE, TM; WIDHOLZER, F.L.; BORNE, B. Va- riagoes nas dimensbes dos ovos de fEmeas de ta- manhos distintos de jacaré-de-papo-amarelo Cai- ‘man latirastris Daudi, 1802. Anais, XX1, XXM, XXIII e XXIV Semana Universitria Gaicha de Debates Biolbgicos, Sociedade de Biologia do Rio Grande do Sul, 1984, p 38-42. ‘THOMPSON, 1.; THOMPSON, M.W. Gendica mé- dica. 4.4. Rio de Janeiro: Guanabara, 1988. TOMAZZI, ML; RIGATTO, M; GOTTSCHALL, CAM. Estudo comparativo de reagtes cardiorres” piratérias em adultos jovens nfo-teinados ¢ tei- nados em face de exercicio moderado, Rew. Ass. Med. Brasil, v22, p.199-201, 1976. VALENTE, VLS.; RUSZCZYK, A SANTOS, R.A. ‘dos. Chromosomal polymorphism in urban Droso- 250 Referencias Bibliogrdficas philawillisioni Rev Bras. Genét, v.16, 0.2, p307- 319, 1993, VARGAS, VME. Avaliagdo de testes de riagem ediag- sto de agentes genotéxices ambientats. Porto ‘Alegre: Curso de Pés-Graduagio em Genética, Uni- vvetsidade Federal do Rio Grande do Sul, 1992. (ese de doutorado.) VERRASTRO, L.; KRAUSE, L. Ciclo reprodutivo de machos de Liolaemus occipitalis Boulenger (Sauris, Tropiduridae). Rev. Bras. Zool, v.16, .1, p. 227-331, 1999. VIANA-MORGANTE, A.M. etal. Premature ovarian failure (POF) in Brazilian fragile X carriers. Ge- net, Mol, Biol, v22, p71-474, 1999. VIEIRA, VG; PROLLA, J.C. Clinical evaluation of eosinophils in the sputum. J. Clin. Pathol, ».32, p.1054-1057, 1979. VITOLA, D. et al. Arritmias primérias em Pediatria, Estudo de 104 pacientes e revisio da literatura. Arg. Bras. Cardiol, vA4, p243-248, 1985. WALLENSTEIN, 8; ZUCKER, C.L.; FLEISS, J.L. ‘Some statistical methods useful in circulation re- search, Circulation Research, v.47, n.1, p.1-9, 1980. WEIDERPASS, E. et al. Epidemiologia do consumo ‘de medicamentos na primeiro trimestre de vida em centro urbano do Sul do Brasil. Rew Sauide Publ, ¥32, 0 4, p.335-344, 1998, WEIMER, T.A. et al. Genetic aspects of Schistasoma ‘manson! infection severity. Rev. Bras. Gendt, v.14, 13, p. 673-639, 1991. WOOLF, B. On estimating the relationship between ‘load group and disease. Ann. Hum. Genet, v.19, p.251-253, 1955, WRIGHT, S. P, Adjusted p-values for simultaneous in- ference, Biometrics, v48, p-1005-1013, 1992. WULFF,A. Caracteristcas aniropométricas: casamen- tos preferencias efertlidade na populagdo de Por- to Alegre, Rlo Grande do Sul. Porto Alegre: Curso de Ps -Graduagdo em Genética, Universidade Fe- deral do Rio Grande do Sul, 1976. (Dissertagdo de rmestrado.) ZAR, J. Biostatstlal analysis. 4. ed. Upper Saddle River — NJ: Preatice-Hall, 1999. Indice A ‘Amostra 17 ‘Amostras 144-152 mostra representativa 144 céleulo do temanho da amostra 146-152 céloulo de n para estimar y 147-148 cleulo de n visando comparar dois grupos 149 cileulo de n visando estimar parimetros 149 {ormulas para o céleulo do tamanho amostral 148- 149 amostras de tamanos diferentes 151-152 amostras de tamanhos iguais 150-151 principsis procedimentos de amostragem 144-146 amostragem aleatia estratifieada 145 mostra estratificada 145 amostragem aleatria por conglomerados 146 amostragem aleatéria simples 14-145, mostra aleatéria simples 144 amostragem aleatria sistemética 145-146 amostras sisteméticas 146 ‘Amostras independentes 68 ‘comparapdo entre duas varitncias 72-73, Aistribuigao F 73, 1 ‘valores critics para um teste bilateral 225-228 valores erticos para um teste unilateral 229-232 Fisher 73 rarho entre as varidncias 73, teste de homogeneidade de varidncias 73 tapas do teste de hipSteses que compara duas mé- igs 70-71 ero padrao da diferenga entre médias amostrais 0 varidneia comum 71 grupo experimental 68 ‘grupo-controle 68, pressuposigdes a0 uso do tester para duas amostras independentes 71-72 signifcdncia estatistice 76-77 teste t quando as varidncias diferem 74-76 i problema de Behrens-Fisher 74 ‘Amostras pareadas 78 ‘amostras pareadas 79 tapas do teste de hipéteses 80-81, pareamento 78 Tepresentago da significincia estaistica de um va- lor ealeulado 81-83 nivel eri amos; ver nivel descr amos- ‘ral nivel descritive amosteal 82 ‘valor ~P; ver nivel deseritivo amastral Atribulos. Ver Varidveis qualitativas e Dados quali- tattvas B Bernoulli 112 Behrens-Fisher 74 Bioestatstica 15 ‘Bonferroni 160-161 ic Chevalier de Meré 111 Conceitos bisicos 14-17 Condarcet 112 CCorrelagao linear simples 84-93 avaliagdo qualitativa de r quanto & densidade 90-91 caleulo do coeficiente de correlaedo em uma amnos- tra 86-87 soma de quadrados 87 soma dos produtos 87 coeficiente de correlagto produto-momento (+) 85- 86 Coeficiente de correlacio de Pearson 85 cocficiente de determinaeto 91-92 iagrama de dispersto 84-85, rama de dispersdo 84 rama de pontos 84 requisitos ao estudo da correlagio 92.93 hhomocedasticidade 92 teste de hipéteses sobre a correlagio 88-89 ‘etapas do teste 89 252 Indice raciociaio do teste 88 transformagao de Fisher para r 88 variagio no coeficiente de correlacio 85-86 D Dados 15 Dados categsricos. Ver Dados qualttaives Dados qualtativos 106-110 sréficos 108-110 ‘artogramas 109-110 diagrama de setores 109 diagramas de colunas e barras 108-109 ‘diggrama de barras 109 ‘diagrama de colunas 108 tabelas de contingSncia; ver iabelas de duas entradas tabelas de duas entradas 106-108 tabelas simples de frequéncias 106 tabelas de entrada simples 106 tabelas de entrada tinica; ver sabelas de entrada simples Dados quantitativos 19.25 distribuigdes de frequéncias (grficos) 22-24 ‘diagrams de bastdes 23-24 agrifico de bastoes; ver diagrama de bastoes histograme 22-2 ogiva 23 Aisttibuigdes de frequéncias (labels) 20-22 tabela de grupemento por intervalo de classe 21- intervaloaberto a diteita 21 tabela de grupamento simples 20-21 ‘eqléocin absolut simples 20 ffeqicia acumulada relaiva 21 ‘fequénciaeelativa simples 20 ‘fequncias scumuladas 20 percent 21 duaris 21 freqdtncia relativa e probebiidade 24.25 Distibuigdo amostral das medias 47-53

Você também pode gostar